You are on page 1of 48

I. The Evolution of Constitutional Law 1. The Founding Era a. The Origins of the Bill of Rights i.

Anti-Federalist Complaints About the Lack of a Bill of Rights

The Anti Federalists complain that the new constitution is defective because it does not protect individual rights. 1. Centinel I, Independent Gazetteer Samuel Bryant of Pennsylvania on the absence of a Bill of Rights from the Constitution, supports it: Bryant has a problem with the fact thatthe PA constitution assures PA freemen freedom from unwarranted searches and seizures, freedom of speech and of the press, while the US Constitution does not. Moreover he points out the fact that the SCOTUS has appellate jurisdiction to both law and fact, which is a novelty in jurisprudence. All this, Bryant says, amounts to a tyranical aristocracy in the Federal Government. ii. Federalist Objections to a Bill of Rights

The Federalists support the Constitution even without the Bill of Rights. James Wilson on the same subject, opposes Bill of Rights: The Federal Government is a government of enumerated rights, and whatever is not granted to it is reserved for the states, and it is unnecessary to enumerate them b/c it's a government of limited powers, and it's dangerous because if they're listed then rights are left out and unenumerated rights have a lower status. Therefore, just because no specific protections are made for such things like the freedom of the press or speech, so long as states protect them under their constitutions, there is no reason to fear that the federal government will impinge upon them because it has on power to do so. A constitution cannot include all powers and rights because there's no space and you can't think of all of them, which is why they are reserved to the state. James Iredell argues the same points as Wilson, adding: Not only is there no space to enumerate all rights, but a dangerous perception may be created suggesting that only those enumerated rights are granted to the people and none other. iii. The Anti-Federalist Reply

Robert Yates, in support of the Bill: States incorporate the protection of the Bill in their constitutions because, supposedly, with the state, what is not reserved is granted, while with the Fed, what is reserved is not granted is reserved. However, this reasoning does not work because the Fed has just as much power to arrest someone and move them around the country as the state does, and everything else related to the criminal process. Basically he's saying that freedom of the press and so on are not implied. iv. James Madison Delivers on the Promise of a Bill of Rights

Madison, in favor of bill of rights:

Madison advises that rights should be sprinkled within the constitution itself, not by adding a separate bill of rights. His argument is that the structural placement of the grant of powers to the branches of government imply that all rights that were present in the states before the Constitution was adopted are retained by the states. Enumerated rights vs. unenumerated rights The argument is that laying out the structure of government and the method of enforcing laws is more important than enumerating rights on paper because enumerated rights are useless without enforcement. The Soviet constitution said you are allowed to criticize the government, but it was a useless right because it wasn't enforced and you weren't allowed to do it. Here, you can question executive action, you can sue your own government, etc. b. The Alien and Sedition Acts The Alien Act gave the president the power to detain individuals who are considered government enemies. The Sedition Act served to quash criticism of the government, and people could go to jail for very little because it was broadly worded. c. The Virginia Resolutions The Constitution matters outside of courts. In England, the Constitution is unwritten, and it constrains only the Crown. Here, it constrains all branches of government. Americans need broad powers to criticize their government, and the Anti-Federalists feel there is not enough power to criticize the government. 2. The Reconstruction Era a. The Thirteenth and Fourteenth Amendments Limiting the Privileges or Immunities Clause The Thirteenth Amendment abolished slavery and Jim Crow laws like the specific crime of black on white rape, special enforcement of involuntary servitude contracts, and vagrancy laws that kicked out black people, as well as the 3/5ths provisions that avoided overrepresentation of southern states in Congress. Under Barron v Baltimore, the Bill of Rights only applies against the federal government. The Bill of Rights uses "no state shall," which is language from Barron in order to directly overturn it and apply due process to states. Due process and equal protection clauses have a broader reach than the privileges and immunities clause. Privileges and immunities are for citizens of the US. The due process and equal protection clauses apply to any person, not just to citizens. The Slaughter-House Cases LA's legislature passed an act that established a monopoly over all slaughterhouses in New Orleans, and gave it to a company. All butchers in New Orleans had to do their slaughtering at this one location after they paid a fee. Petitioners lose on all counts.

[OVERTURNED] The 14th amendment establishes two types of citizenships, national citizenship with its own set of rights, and state citizenship with a separate set of rights, and the privileges and immunities clause only applies against the federal government. These rights include the right to land at national ports, protection on the high seas, petition the federal government, and so on. The claims in this case are under state citizenship and do not fall under the privileges and immunities clause. This is a bizzare concept because the Civil War was not fought for the right to land at national ports, it was fought to cast off oppression by any government. The 14th amendment was meant to overturn Dredd Scott, which asserted that any person of African descent, whether free or not, was not and could never be a US citizen. Here, no African slaves were involved, only LA butchers, so they can't claim rights under the fourteenth amendment. y [DISSENT, CURRENT CONTROLLING LAW, NOW DONE THROUGH DUE PROCESS CLAUSE] The 14th amendment protects citizens from their state legislatures through the privileges and immunities clause, which is defined in Civil Rights Act as the right to "make and enforce contracts, to sue, be parties and give evidence, to inherit, purchase, lease, sell, hold and convey real and personal property, and to full and equal benefit of all laws and proceedings for the security of person and property." The right to one's own labor is one of the most funamental rights possessed, as mentioned in Smith's Wealth of Nations. Bradwell p 281 Married woman wanted to become a lawyer but couldn't b/c laws of coventure made her contracts void. [OVERTURNED RULE] The privileges and immunities clauseprotects only those rights that are deemed to be federal, not state rights. Right to practice law is not a right of national citizenship that is protected by the federal government because it is not like the right of protection on the high seas or the right to petition the federal government. Today this would be argued under equal protection clause. iii. Limiting the Amendment to Racial Discrimination Cruikshank A black militia took over a church in protest of a local election, and a party of white democrats set the building on fire, shot blacks who were fleeing, and took the rest down to the river for execution. D was one of the white people who is defending his lynching. [OVERTURNED] The right to peacefully assemble is a natural right that existed before the Constitution was enacted, and was not signed away by the people, but the 14th amendment operates only on the federal government not on state governments or private parties. Here, it was a private militia that infringed on the right to bear arms and to assemble peacefully, not by the federal government or even the state government. Strauder A black man is on trial for murder in WV, but a statute forbids black persons from participating in jury duty, so his jury was all white. An all white jury for a black person is a violation of the fourteenth amendment because that amendment was meant to give blacks all the same rights as whites.

The court relies on a liberal interpretation of the fourteenth amendment by considering the "spirit and meaning of the amendment," which was novel at the time. This is still good law, but now under equal protection. Court would have used privileges and immunities if it were applicable to states, but it wasn't under Slaughterhouse. iv. The Amendment Does Not Extend the Bill of Rights to the States Presser Presser, armed with a sword, rode on horseback in Chicago leading 400 men armed with unloaded rifles to protest police violence. He was arrested and fined for assembling a militia and drilling inside the city. Right to bear arms, again, only applies against the federal government, not city or state. It is a right of national citizenship, not state citizenship. Here, D did not belong to any national militias, and he was stopped by state and city powers, not federal powers. Note that this is still good law, and the second amendment has not been incorporated against the states, but the SCOTUS can do that if it wishes, and most people expect them to do so. v. Limiting the Amendment to State Action The Civil Rights Cases Theaters and other places of public entertainment refused to seat black people. [OVERTURNED] Although the Constitution guarantees the protection of civil rights against state action, it does not protect civil rights against private action. Here, it was theaters and other private venues who were discriminating, and the state was not complicit. An individual cannot violate someone else's civil rights, that's a private wrong that police handle. [OVERTURNED] Thirteenth amendment deals only with enslavement of a person, regardless of that person's race. No enslavement is claimed here. [OVERTURNED] Segregation and slavery are different things. The thirteenth, fourteenth and fifteenth amendments deal with slavery, not with segregation. Dissent: HARLAN: Blacks have social burdens and stigmas placed upon them as a result of their previous enslavement, and the source of law is found under the Civil Rights Act of 1866. The adoption of the civil war amendments aimed to do more than to emancipate slaves, but to bring black people to the same level as whites. Hotels and inns have to apply for state licenses, and there's enough of a government link there to disallow discrimination in those settings. So far the 14th did not incorporate the 5th against the states, only protects those few federal citizenship rights, and y the enforcement clause (second clause) does not enforce 14th amendment against private action. y The 13th amendment empowers congress to act against private action, but not all racially discriminatory action, rather it has to add up to an incident of slavery or other involuntary servitude Moving on to 14th amendment, and how it applied 5th at the turn of the century.
y

vi. Facially Neutral Laws

These are laws that are neutral on their face but in these two cases are discriminatory in the way they are applied. YickWo City of San Francisco passed legislation banning laundries in wooden buildings without permits. Two thirds of laundries were ran by Chinese persons, and all but one of them who applied for permits were denied. A law that is facially neutral can be challenged when it is discriminatory in its application. While the statute here is not discriminatory on its face since it regulates safety, it is discriminatory in application because supervisors do not give any reason for denying Chinese laundry operators licenses to do their job other than because it is their will. They apply the law with an "evil eye." Note, however, that the facts must be egregious, and the state must give no reason for their discrimination. This case is out there in the spectrum. Also, the fourteenth amendment applies here because it does not limit itself to protecting only citizens because it says "all persons." vii. The "separate but equal" doctrine Plessy LA passed a segregation law separating whites from blacks on passenger train cars. Plessy wanted to be seated in a white coach, claiming he is white (1/8 black 7/8 white). [OVERTURNED] While the 14th amendment was meant to give equal political rights to whites and blacks, it was not meant to give equal social standing to the two races. The court bought the state's justification that the law was in place for the public good because it established "usages, customs and traditions of the people." This establishes how weak state reasoning has to be for a law to be upheld. "No reason not to" is not good enough, but any other justification works. HARLAN dissent: There is no caste system in the US, and one race cannot subjugate the other. These statutes are designed to stamp inferiority on the black race, which is counter to the intent of the 14th amendment. Government has to be color blind in making certain decision even if it means inquiring into the real purpose of a statute. When does a court go "behind the statute" of a facially neutral law and into the intent in application? In Plessy, Harlan went behind the statute, in YickWo the court did not. [My opinion: look at the facts of the case, and at the totality of the circumstances.] 3. The Progressive Era Court a. The First Amendment Schenk Soldiers began mailing pamphlets urging other soldiers to assert their rights and oppose the draft. The military prevented the transmission of those pamphlets through the Espionage Act. A person may not falsely shout "fire" in a crowded theater, or utter any other words that have the effect of force. If words create a clear and present danger, Congress has a right to prevent that speech.

ANALYSIS for proximity and intent, which allows gov to curb speech: What is the danger, and how close is it? In Schenk, nation at war was sufficient. What is the intent of the speech? Here, the nation is at war, and this speech hinders the war effort and recruitment efforts and puts the lives of other soldiers at risk. The danger is great and close. It is broad though. Debs Leader of the socialist movement was indicted under the Espionage Act for giving a fiery speech to troops encouraging them to interfere with the recruitment process. D was found guilty and sentenced to ten years. If part of the intent of the words is to encourage those present to obstruct the recruiting service, the speech may not be protected. Here, the defendant admitted that he is against war, and that he praised those who stood in the way of the draft. Proximity and intent are met. Abrams Ds printed pamphlets drafted to urge the curtailment of production of arms and ammunitions needed for the war. The Ds were Russian nationals who proclaimed to be anarchists. If the language was meant to provoke forceful opposition to the war effort, it is not protected speech. Here, court holds that the proximity and intent test is met and the speech is curbed. [DISSENT, MORE IMPORTANT] First amendment protects a marketplace of ideas so long as speech is not like falsely shouting "fire" in a crowded theater. HOLMES: Where the proximity and intent test is not met, the marketplace of ideas should weed out powerful thoughts, and the truth of a thought is tested by its ability to gain a foothold in that marketplace. Gitlow D was convicted of criminal anarchy. He is challenging the criminal statute as violative of the 1st amendment as applied to the states through the due process clause of the 14th amendment. He was a member of the Left Wing Socialist Party, and he printed a manifesto advocating the overthrow of the government. The state has the right to self-preservation and therefore has the right to punish speech that advocates the unlawful overthrow of government. Here, the overthrow of government can only be accomplished through violent means as described in the manifestos D circulated. [DISSENT] Holmes argues that every idea is an incitement, it's only a matter of tone. The proper test to apply is proximity and intent. Here, the proximity and incitement test is not met because it is unlikely that the danger would come about. Stromberg Statute proscribed the display of any flag, banner or badge anywhere as a sign in opposition to organized government (anarchism). A statute that is vague because it's unclear what it covers and overly broad because it includes legal behavior is unconstitutional.

4. The Warren Court a. Racial Discrimination i. The Equal Protection Clause Brown v Board of Education After the abolition of slavery, Jim Crow laws, or the "black codes" systematically excluded africanamericans from accessing public institutions like equal education, marriage, and voting. In Kansas, SC, VA and DE, schools were segregated and did not accept africanamerican children. Lower courts denied access based on "separate but equal" doctrine announced in Plessy. Separate cannot be equal because separation implies inferiority. The court relies on social science studies in footnote 11, which is controversial because the courts may not understand those studies, and they may not support their point. Education is the basis of our democracy because it is required for performing the most basic of public responsibilities like voting and holding informed discussions about politics. [HARLAN] Scientific studies are not necessary to debunk separate but equal, the explanation is that the country has evolved in its thinking on race relations. Note that there is an argument that Brown did not overturn Plessy because Brown is about education, which is necessary to the basic right to vote, while Plessy is about transportation. My opinion: Obviously if Brown was decided under Harlan's opinion the justificaition is not the scientific study, and Brown would abolish all segregation. If looking at the court's reasoning, one can still argue that transportation can be linked to any number of basic rights under the constitution as central as voting, like the right to earn a living. Since no other transportation was available at the time, public transportation would be important. Counterarg is that colored people could still ride the bus, just that they had to sit in the back. Then you say that the psychological effects of segregation are serious for adults as well as children, which was the reasoning of the court in Brown. SCOTUS applied it to all segregation decisions: Brown made segregation illegal under the equal protection clause because separate is inherently unequal. Note that the court did not find it necessary to apply the due process clause. The holding is narrow because it doesn't explicitly recognize educaiton as a fundamental right, but it is also broad because this case is applied to other settings besides education. ii. The Due Process Clause of the Fifth Amendment

Bolling v Sharpe Same facts, but in D.C., which is federal, and due process clause of 14th doesn't apply, so Ps claim is under 5th amendment. The court reverse incorporates due process of 14th to 5th. The due process clause of the fifth amendment includes the equal protection clause of the fourteenth amendment. OTOH you can argue that the court was result driven, but that is a negative for constitutional law. It questions how much Brown was about equal protection, if the result was more important than the method. Here, however, it's ok to be result driven to some extent. Academic discussion where issue is whether Brown or Bolling'sjustificaitons are correct, i.e. whether cases should be decided on originalism or through social study justifications:

McConnell Defends the minority view that Brown was actually decide through originalist meaning, b/c Republicans at the time of the passage of the 14th amendment were against segregation. iii.

Klarman Defends the popular view that Brown was based on newfound social studies that showed black children were adversely affected by segregation, not on the original intent of the 14th amendment.

Racial Classifications Applying Equally to Whites and Others

This is current doctrine, and applicable law. Loving v Virginia Virginia passes a statute forbidding interracial marriages. A black woman and white man from VA married in D.C. and returned to VA. Judge below said God separated the races geographically, and that's proof of God's intent that they should not comingle. He sentenced them to a year in prison but he suspended the sentence so long as the two left VA and did not return for 25 years together. Equal protection violation? Mere "equal application" of a statute containing racial classifications is not enough to exclude that classifications from the 14th amendment's proscription of all invidious racial discriminations, and it triggers strict scrutiny. Strict scrutiny can only be overcome through a showing of a compelling and permissible state interest independent of racial discrimination. ANALYSIS for whether a right is fundamental and gets you strict scrutiny (part of standard of review ANALYSIS, see below under equal protection): 1. What is the nature of the right being asserted? 1. If it's a fundamental right, you're likely to get strict scrutiny and the statute will likely fall. 2. Is the right asserted "deeply rooted in tradition and history?" Would we find the right to marry outside of your race "deeply rooted in tradition and history"? 2. What is the government's justification for its policy? 1. What if the reason for the policy is not white supremacy but to maintain the integrity of the family?

iv.

The Enforcement Clause of the Fourteenth Amendment

Katzenbach v Morgan p 574 The Voting Rights Act of 1965 enabled Puerto Ricans to vote b/c it said anyone who received education through 6th grade in a Puerto Rican school could vote in NY. New Yorkers attacked the constitutionality of that statute that came in conflict with a NY law requiring all voters to be able to read and write. Does the COTUS have the Constitutional power to pass such legislation under the enforcement clause of the fourteenth amendment (section 5)? Katzenbach incorporates the McCullouch standard for deciding the scope of congress's power under section five of the fourteenth amendment.

This is an expansive standard that basically asks whether a law is plainly adapted to secure the rights it seeks to secure. Appropriate legislation, under McCullouch standard of enforcement clause: y Has a legitimate end, y is within the scope of the constitution, y and all means are constitutional that are plainly adapted to that end, y are not prohibited, y but are consistent with the letter and spirit of the constitution.

II. Equality 1. The Equal Protection Clause a. Introduction to Equal Protection ANALYSIS for Levels of Scrutiny: y Is this a suspect classification or does the law burden a fundamental right, or is there viewpoint discrimination? (ex.: racial classifications, right to vote [Harper], travel [Saenz]). ANALYSIS for whether a right is fundamental and gets you strict scrutiny: 1. What is the nature of the right being asserted? 1. Is the right asserted "deeply rooted in tradition and history?" Ex.: ould we find the right to marry outside of your race "deeply rooted in tradition and history"? Probably not, but if you phrase the question of whether you have the right to marry period, then it comes out differently. 2. What is the government's justification for its policy? 1. What if the reason for the policy is not white supremacy but to maintain the integrity of the family?  If Yes, then strict scrutiny is applied, and the government has the burden of proof to show: y A compelling state interest y Means that are narrowly tailored to accomplish that end y Strict scrutiny is often called "strict in theory but fatal in fact" because it almost always strikes down a statute, but ocasionally it doesnt (Grutter, Korematsu).

If No, then is it a semi suspect classification? (ex.: gender [Virginia Military Institute], you have to bench more than 160 lbs, etc.)? y If Yes, then some form of heightened scrutiny such as intermediate scrutiny, and the burden of proof is still on the government to prove: y An actual and important purpose which is less than compelling but more than legitimate y Means must be substantially related to the end
y y

If rational basis "with bite" applies (Sexual orientation, etc.): y Evans, Cleburne, Romer

If deferentialrational basis applies:

The burden of proof is on the challenger, and they must show: y The government interest is not legitimate or not rationally related to a legitimate interest. Note that any state interest is good enough to justify a law, even if the attorneys come up with the reason at trial. y Cases include Railway Express, Lee Optical y Ways in which a law can fail under rational basis: What is the interest that the legislation is trying to serve and the classification they have used to try and serve the interest? Over-inclusive An over-inclusive statute bans more people that you need to ban, so you're harming more people, which is more likely to be taken care of in the political process than legal process. If you declare that methodone clinics dont work and challenge it as unconstitutional, but in fact some people do well in methodone clinics and get better, then it's over-inclusive. Under-inclusive Legislation bans some advertising on trucks so that people don't crash their cars when they're driving on the road, but they don't go after all the truckers, then it's under-inclusive (Railway):
y

If No, does it infringe on a fundamental right? If yes, see above analysis for assessing fundamental right violation. y If No, then rational basis applies.
y

Railway NY statute banned the practice of advertising on the side of commercial trucks for purposes unrelated to the actual business of the particular truck. State's justification was that it diverts the attention of drivers and causes traffic problems. D argues that the regulation is underinclusive because it bans advertisements for businesses unrelated to the truck's business, but not if the business advertised was also the business of the truck. Petitioner argues that it does not matter whose business it is when it comes to the traffic problmes in real life. Equal protection violation? No, statute stands under rational basis review. Equal protection does not require that a statute ban all problematic advertising or none at all, so long as it bans some problematic advertising. SCOTUS defers to the judgement of the legislature whether some advertising is more dangerous than other advertising, so long as the interest is legitimate and rationally related. b. Rationality Review Generally Lee Optical An act was passed that allowed only Optometrists, who are M.D.s to make lenses for glasses, and not opticians who did not obtain this higher medical degree. Not constitutional under due process, statute fails.

The state may regulate activities that bear on the health and welfare of the public, but if activities can be broken up into some that bear on health and welfare and others that do not, the state cannot regulate all of them becaues they are in close proximity to each other. Here, eye safety is within police power which is why they can regulate ophtalmologists, but the job of optometrists is to fit lenses, which does not bear on health or safety. c. Equal Protection of Fundamental Interests Harper VA imposed a poll tax for voters. Statute fails under equal protection, no poll tax b/c wealth is a suspect classification. A voter's wealth is not a valid qualification for the purpose of voting just like race, creed or color. Voting is a fundamental right, and wealth as a qualification is a suspect classification, so it fails. San Antonio v Rodriguez In TX, property taxes were the primary determinant of how much money was spent on students in a given school district. In one district that is mostly Mexican-American and African-American, low incomes produced low property taxes and resulted in $356 per student, whereas in the wealthy district high property taxes and incomes resulted in $594 per student. Lower court found that education is a fundamentalright and wealth as a classification was a suspectcategory, so it struck down the disproportionate funding system because the state could not provide a compelling state interest. Wealth is not a suspect classification for purposes of public education, disparity in education stands, tax based student expenditure stands. Where a state statute discriminates on the basis of a suspect classification, which is usually on the basis of a fundamental right, a court will apply strict scrutiny. The equal protection clause is violated only when a person is completely unable to pay for such a state service and is therefore precluded from it, unless that right is a fundamental right such as voting, which education is not. Here, petitioners are not excluded from the TX educational system, they are just forced to go to schools that expend fewer resources per student. This is not like the voting case or other similar cases because in those cases the petitioners were totally unable to pay for the service and could not exercise their right to vote. Some education is not no education. d. Racial Discrimination & Strict Scrutiny Affirmative action programs are designed to even out the playing field for underrepresented minorities. The main POLICY is to remedy prior injustices, or to counteract unconscious biases. Current doctrine: 1. Skepticism All racial classificaiton get the highest possible scrutiny. 2. Consistency Whether it is invidious or benign classification 3. Congruence Fed gov and state and local govs get the same treatment with respect to equal protection.

There are two policies, color blindness and antisubordination. Regents of UCal v. Bakke Applicants to the US Davis med school had to have a 2.5 GPA or above to get in, but if they opted to be considered as minority students (which were African Americans, Mexican Americans, Asian Americans and Native Americans), the GPA cutoff was relaxed. Rejected white student filed suit under equal protection. Diversity requirement unconstitutional, white student wins and is admitted. Racial and ethnic distinctions of any sort are inherently suspect and thus call for strict scrutiny, regardless of who is the opressor and who is the oppressee. Countering the effects of societal discrimination and segregation is not a compelling state interest for strict scrutiny (OVERTURNED in Grutter below). "The clock of our liberties, however, cannot be turned back to 1868." Racial classifications for constitutional purposes cannot "vary with the ebb and flow of political forces." Racial diversity in a medical school setting is a compelling interest because it brings diverse experiences, outlooks and ideas that enrich the training of its student body, but race may only be one factor in the admission decision. In its implementation, UCal must follow Harvard's example and allow for ethnicity to be only one of many considerations in an application. This is because race is not the only element of a person's makeup that brings diversity to campus. Wealth, geographic origin and lifestyle can be other factors. Grutter v Bollinger (grad school admission process, const b/c color blind) Michigan law school seeks to achieve a "critical mass" of students whose "racial and ethnic diversity have been historically discriminated against, like Afrcian-Americans, Hispanics and Native Americans who without this commitment might not be represented in our student body in meaningful numbers." P is a white Michigan resident with a 3.8 and 161 who got waitlisted and rejected, and is suing under 14th, civil right act and state laws, claiming that race is a predominant factor in the admission process. Law school admission process is constitutional under equal protection, white student loses and is rejected. Student body diversity to remedy the racial discrimination in the past is a compelling state interest that can justify the use of race in university admissions. Michigan law's admission policy passes the Bakke test because it does not require a set percentage of minorities to be admitted. It looks to a "critical mass" that is defined by the "educational benefits that diversity is designed to produce." Note that this is a case where strict scrutiny is applied but law passes anyway because countering effects of racial discrimination is a compelling state interest. (this is the only case besides Korematsu [strict scrutiny overcome for national security purposes] where this happens). An admission process may not employ racial quotas or unduly burden individuals who are not members of the favored racial group, but may use race as a plus in the candidate's application without insulating the individual from comparison to other applicants of different races. Michigan law's admission policy is narrowly tailored even if it takes numbers into consideration. Paying attention to numbers is not automatically a quota system because the admission process remains flexible since it counts race as merely a plus on the application and considers the application as a whole. In 25 years, affirmative action will no longer be needed. The court pulls this out of the air. Gratz v Bollinger (undergrad admission process, unconst)

Michigan undergrad's admission system allocated points according to an applicant's attributes, and ranges b/w 100 to 150 were a sure thing, below that it got less sure, and below 70 the applicant was rejected. Artistic talent earned a candidate a maximum of 5 points, while classification in a traditionally underrepresented minority earned 20 points automatically. Admission is unconstitutional under equal protection b/c strict scrutiny is applied and admission policy is not narrowly tailored. Each particular candidate must be considered individually, assessing all of the qualities that individual possesses and what that individual can contribute to the institution. Here, the rigid quota was not an individual consideration of each application, because for example if a student's artistic talent rivaled that of Picasso, that applicant could only get 5 points, but if an applicant race was mechanically determined to fall under the underrepresented category, 20 points were automatically granted towards that applicant's admission. In close cases, race was always a determining factor. Bottom line: In Grutter, racial classification was ok, while in Gratz it wasn't. The college admission program was too mechanical and it didn't look at multiple factors, the law program did. You can take racial classification into the calculus, but you can't give it unique weight. Diversity cannot be defined as race alone. Parents Involved (antisubordination, const) School Dist decided that in order to improve diversity, they would start bussing black kids into the white schools. The School Dist looked at the racial composition of a school and figured out a way to move people around to meet certain quotas. Scheme is unconstitutional, race can't determine where kids go to school. Any discrimination on the basis of race summons strict scrutiny that can only be overcome by showing a compelling state interest. There are two compelling interests in education: 1. The first compelling interest that overcomes strict scrutiny is racial balance in schools to remedy the effects of past intentional discrimination. Grutter However, here, the Seattle school district had never been segregated historically, and was never subject to desegregation decrees. 2. The second compelling interest that overcomes strict scrutiny is the interest in diversity of backgrounds and opinions in higher education of which race is merely one component. Here, the scheme is unconstitutional because the distribution process is mechanical, and race is the only component in the decision. Race is also defined as either black or white. "The way to stop discrimination on the basis of race is to stop discriminating on the basis of race." Adarand v Pena (current doctrine) The Small Business Act created federal incentives to hire subcontractors who qualified as businesses hiring "socially and economically disadvantaged individuals." Adarand submitted the lowest bid for a job, but Gonzales got it because the contractor would receive a payment from the federal government for hiring a qualified minority-employing subcontractor. Unconstitutional (remanded), act fails, white subcontractor gets job. Racial classifications are subject to strict scrutiny regardless of whether they have a benign intent, because what may appear to be benign intent may in fact be odious intent. Governmental racial classifications are judged by three principles: a. Skepticism:

Any racial preference in the law receives a "most searching examination." b. Consistency: Same as skepticism, but more specific. Racially based laws receive the standard of review under the equal protection clause resamegardless of whether they are benign or odious. c. Congruence: Federal law and state law are treated the same for the purpose of the equal protections clause (this is not the law, no majority). Washington v Davis "Test 21" tested applicants' verbal and written communication skills for those who wished to enter the D.C. police academy. African American candidates scored lower on the test and were denied entry as a result than white applicants proportionally. Test does not violate due process/equal protection, African Americans denied. An official action will not be held unconstitutional solely because it results in a racially disproportionate impact. Plaintiff must show both discriminatory purpose besides discriminatory impact. The state's actions are intended, and a discriminatory purpose exists, if the consequences of the state's actions are reasonably foreseeable and the consequences are desired by the state. Here, the exam tested written and oral communication skills, which are important for police officers, so the test was neutral on its face. The purpose was to have a well qualified police force in D.C., so purpose was not discriminatory. The fact that African Americans failed it disproportionally was not enough to show that the test systematically weeded out African Americans. ANALYSIS for discriminatory purpose: a. Is there overwhelming statistical proof whereby no other explanation makes sense besides discrimination? (YickWo, this rarely happens anymore) b. Is there a history of discrimination? (Univ. of Mississippi requirement to get letter from alum). e. Sex Discrimination & Intermediate Scrutiny Frontiero v Richardson Woman in the military wanted to claim her husband as a dependent to place him under medical and dental coverage offered by military, but only a man could claim a woman as a dependent whether she depended on him or not, but a woman could only claim a man if he depended on her for at least half of his financial support. Equal protection violation. When two individuals are otherwise equally entitled to a legal right, the male applicant cannot be preferred over the female applicant. Sex classifications are inherently suspect and must be subjected to strict judicial scrutiny. (This case rachets up scrutiny from rational basis to a form of heightened standard, but not to strict scrutiny b/c only four justices sign onto Ginsburg's opinion). The rationale is b/c you're born with your gender, it is mostly immutable, and it cannot be hidden. Here, court applies strict scrutiny for sex discrimination, but this is not the standard that it ultimately adopts for sex discrimination cases. It's also a plurality, so not really controlling law. OTOH the fourteenth amendment was not passed because of gender discrimination. Craig v Boren adopts intermediate scrutiny

OK law prohibits the sale of "nonintoxicating" beer with alcohol content of less than 3.2% to men under 21 and women under 18. This violates equal protection clause. Classifications by gender must serve important governmental objectives and must be substantially related to achievement of those objectives. The court did not classify sex discrimination as a suspect category, but rather "intermediate scrutiny" which requires "minimum rationale." US v Virginia p 978 (rational basis scrutiny with bite) Virginia Military Institute is a public school that employs a military style type of discipline modeled after old English boarding schools. It produces individuals who are better able to deal with stress and adversity in life through its hardcore program. Business leaders, members of congress and other noted individuals have graduated from this school, and they attribute their success to this education. The style of education is not particularly unsuited for females, but it is a male school. Women get rejected. This is a violation of the equal protection clause. Parties who seek to defend gender-based government action must demonstrate an "exceedingly persuasive justification" for that action. Sex classification mays may be used to compensate women for gender specific disabilities, but not to create or perpetuate the legal inferiority of women. The standard of review is rational basis with bite, and the court demands an exceedingly persuasive justificaiton which it does not get here. Explicitly sex based classifications are not allowed, but note that these are rare nowadays. So sex classification can only be justified if it's positive, not negative. Here, the government argues that VMI provides educational diversity to the state. Court agrees, but does not agree that this is the fundamental reason why the program exists, rather that is more like a positive side effect. The state next argues that admission of women would greatly affect VMI in that they would have to build separate barracks for women, and men might not want to attend VMI. The court grants that, but says that VMI would not be altogether destroyed, so the reasoning is not "exceedingly persuasive." Sacrifices must be made. Dissent, SCALIA: This decision destroys single sex education The three levels of scrutiny, "rational basis," "intermediate," and "strict" scrutiny are not scientific tests, and they are outcome determinative. Once the court picks the standard of review, the outcome is mostly a given. f. Other Discrimination & Heightened Rational Basis Scrutiny Moreno (heightened rational purpose/raitonal purpose "with bite") The Food Stamp Act did not allow beneficiaries to share their food stamps to person who were unrelated to them. P lived with a family who received food stamps. Although not related, she lived under the same roof, shared food costs and she helped raise their children. Denial is not rational, so it violates equal protection, unrelated woman gets to share in food stamps. A challenged classification must rationally further some legitimate government interest, whether mentioned in the legislative history or not.

Here, the only justification the government could come up with is to prevent hippie communes from sharing food stamps. The court finds that harming a group for political reasons cannot be a legitimate government interest. Note that in this case, the court looked to the government's actual purpose and judged whether it is legitimate and rationally related, which is the "heightened" rational purpose test (unlike in Fritz).

g. Rational Basis Redux Fritz (pure rational basis) A railroad that was operated by the government had a retirement scheme in place that awarded more retirement benefits to those who worked more than ten years and had a "current connection" with the rairoad industry, as opposed to those who worked more but no longer had a connection with the railroad industry. The retirement scheme is constitutional under equal protection, and some are denied benefits. Legislation regarding economic benefits must have a fair and substantial relation to its object, but it will not be overturned simply because it is unwise or unartfully drawn, and proof of Congressional intent is not required. Here, COTUS drew lines on who gets what benefits in order to effectively phase out retirement plans. Here, the government's justification is that those who are "career railroaders" deserve more benefits than otherwise, and that's not an arbitrary line to draw. This is true rational basis review because the court does not look at the actual purpose, but a hypothetical purpose is sufficient (unlike in Moreno). Rational basis means that the means have to be rationally related to a legitimate government interest. The purpose is not an "actual purpose" but any rational purpose. Rhenquist says that legislators don't always know why they're passing legislation, and more statutes would get struck down if they don't achieve their actual purpose. Cleburn City in Texas denies a permit application for building a home for the mentally retarded. Unconstitutional under equal protection heightened rational review (below gender standard though), home for mentally retarded has to be built. Where individuals in the group affected by a law have distinguishing characteristics relevant to interests the state has the authority to implement, the courts will only impose the heightened standard of review. Individuals who are mentally retarded are immutably different, and the state has a compelling interest to treat them differently because they have different needs. The court overturns the classification of the mentally retarded as quasi-suspect, but it affirms the outcome (reversed, home can be built) on the ground that the state's justification does not hold up under rational basis review. All concerns the state lists for building the mentally retarded house can be brought against any other nursing home or hospital that could have been built there without the requirement of a special permit. This implies that the standard is slightly above rational basis even though it's called the same thing, b/c rational basis always fails. OPINION: When a feature is immutable, whether a state can treat it differently or not depends on whether it makes sense to treat it differently. Black/white never makes sense, because needs are the same regardless of color. However, for gender sometimes it makes sense like in the case for bathrooms, or education (what's the difference between the need for privacy and the need not to be distracted by an attraction/repulsion to the opposite sex at school?)

Romer An amendment to Colorado constitution dissallowed protected status to individuals based on their sexual orientation (gays can't be minorities) that would entitle them to preferences in quota systems. Amendment is unconstitutional under equal protection, gays get preferred status Where a law is passed for no reason other than to harm or deny access to law for a group of people, even if it's rational basis the law will be struck down. The court looks for rational relation and didn't find one (so they applied rational basis standard here). They didn't make sexual orientation a suspect classification however. Note however that the fact that gays were historically discriminated against plays a role in the decision. 2. The Privileges or Immunities Clause Saenz v Roe CA enacted a statute limiting the maximum welfare benefits available to newly arrived residents to the benefits they would have received in their prior state. It's a constitutional issue b/c CA receives federal reinbursements for payments. The court didn't reach the merits that time b/c Secretary invalidated it, but then COTUS enacted PRWORA that had the same effect. Unconstitutional under equal protection, CA has to give new residents the same rights. The right to travel, treated as welcome by another state, and treated the same as other residents of that state is a fundamental right under Shapiro, from Art IV section 2 Privileges or Immunities clause, and will receive strict scrutiny review accordingly. The statutory scheme discourages travel, even if it does so incidentally, and for that reason it's unconstitutional. If CA is trying to discourage travel for the purpose of gaining welfare benefits, empirical evidence shows that few people do this b/c CA cost of living is high, but even if that's true, it's stilunpermissible under Shapiro. The right to travel includes: y The right to enter and leave a state. Undesirable people cannot be kept in or out. y The right to be treated as a welcome visitor, not as an unfriendly alien This is under the privileges AND immunities clause (in art IV section 2, different from priv or immunities). When you come to a state, you have certain privs and imms just b/c you're a resident in a sister state. The counterargument is that you have to be a resident to go to school, but not for welfare? Case's reasoning is that welfare benefits are portable while education benefits are not. This is questionable reasoning but this is what the case turns on. 3. State Action Under the state action doctrine, constitutional limitations apply only to state conduct and not to purely private activities. The government must do something before the constitution is violated. The word "state action" is broad here, it means government, state or federal. Receiving government funds is not enough (Ex.: if you speak out at Villanova against a professor, and the Dean kicks you out, there's no 1st amendment claim b/c Villanova is a private actor). Generally issues arise where a private party performs a "public function." DeShaney State inaction is not sufficient for a first amendment claim.

Young boy is in custody of father after divorce, and his father abuses him. Social service agency get reports but do nothing about it. The abuse is so bad that kid gets permanent brain damage and goes to a state institution for life. Mother claims that his liberty was taken away w/o due process b/c social services did nothing once it got the reports. Court rejects the claim. Marsh P is a Jehova's Witness distributing religious literature on the premises of a company owned town contrary to town management's desires. The shopping center is accessible by anyone, except that the land is owned by the corporation. She claimed freedom of speech under first and fourteenth, but lower courts ruled against her b/c town was owned by Corporation and they can do what they like with their property. She was arrested by the sherriff under a state statute and thrown in jail Facilities that are built and operated primarily to benefit the public, and whose operation is essentially a public function, those facilities are subject to state regulation. The company's right to exclude people from its property wasn't determinative b/c the more a company opens up its doors to the public as a public function, the more it's exposed to constitutional claims. Running a company town is very close to being a public function. there is more here, but it's not an essay question, so check under "state action" for further details and cases.

EXAM: State action cases are fact specific, so you need to look at cases. you won't see a question about state action as essay, only as multiple choice. It may come up as a side issue on an essay question.

III. Liberty 1. Freedom of Speech & Press General rule is that content based restrictions on speech receive higher scrutiny than content neutral restrictions: Content - Based receive greater scrutiny No protests in the park No political posters on public bus shelters No public protests of abortion No public burnings of the American flag. a. What Is Speech? i. When Is Conduct Speech? Content - Neutral receive less scrutiny No loud speech in the park No posters on public bus shelters No noisy speechwithin earshot of hospitals No public fires.

US v O'Brien p 1086 D and 3 others burnt their Selective Service registration certificates on the steps of the South Boston Courthouse. A crowd gathered, FBI took him into the courthouse where he admitted he burnt the certificates in violation of federal law. The Military Training and Service Act forbids the knowing mutilation or destruction of the certificates. D can be arrested for mixed speech nonspeech activity where there are sufficiently important governmental interests at stake. Where legislation that pursues a sufficiently important governmental interest has only an incidental effect on speech, that incidental effect will only receive intermediate scrutiny. EXAM: 4 part "O'Brien" test for intermediate scrutiny, only for content neutral laws: 1. If ends of legislation are within constitutional power of government (police powers) 2. If it furthers an important or substantial government interest Here, national security, ability to raise an army quickly and communicate to draftees. 3. If the government interest is unrelated to the suppression of free expression (content neutral?) a. If it is related to free expression, then scrutiny is applied Here, the government interest is unrelated to the suppression of free speech because the reasons are administrative efficiency, a fast and efficient way of raising an army for changing circumstances etc. 4. If the incidental restriction on alleged First Amendment freedoms is no greater than is essential to the furtherance of that interest. Speech cannot be a limitless variety of conduct just because a person intends to express an idea. The Military Act meets the requirements of being sufficiently important, and since the activity was mixed speech and nonspeech D can be arrested and tried constitutionally. P argues that once the card is received, he has notice and the card serves no further purpose. It doesn't serve a purpose anyway b/c he does not ned to be told his address and physical characteristics. Court says that the cards have other pusposes like acknowledging that P registered for the draft, communication b/w registrants and local boards, etc. The interest is substantial b/c gov needs a fast and efficient way of raising an army for changing circumstances. Texas v Johnson p 1089 D participated in a political demonstration in Dallas in the wake of Reagan's reelection where he burnt a flag and said "America the Red White and Blue we spit on you." He was convicted for flag burning, not for his uttering. The conviction does not stand, flag burning ok, because it's entirely communicative. An act is communicative and counts as speech if there is an intent to convey a particularized message, and whether there is a likelihood that the message would be understood as communicating a message. O'Brien's rule did not apply here because the burning of the flag was entirely communicative, it was not mixed communicative and action. The flag is a symbol, and there is no state interest countervailing the burning of flags. In O'Brien, there was a state interest against burning draft cards that had to do with efficient communication with draftees and the ability to raise an army quickly. Also, the "fighting words" rationale did not apply because the protest was peaceful and there was no likelihood of injury.

First, the court must decide whether action is expressive conduct. An action is expressive conduct if an intent to convey a particularized message is present, and whether the likelihood is great that the message would be understood by those who view it. Such ations include students wearing armbands to protest Vietnam, blacks sitting in whites only areas to protest segregation. If activity is expressive conduct, then D can bring a 1st amendment defense, and the court will have to a. decide whether the state's regulation is related to the suppression of free speech. b. If it's not related to the suppression of free speech, then the less stringent O'Brien standard controls, but if it is, then a more demanding standard applies. The government may not prohibit the expression of an idea simply because society finds the idea itself offensive. Here, the conduct was "sufficiently imbued with elements of communication" because a) TX conceded that the burning of the flag was communicative, and b) the flag was burnt as the culmination of a political demonstration just as Reagan was elected. Now the court has to look at whether the regulation is related to the suppression of free speech. TX's justification for the statute is that it preserves the peace and the flag as a symbol of nationhood and unity. The first interest is not implicated, and the second is related to the suppression of speech. No disturbing the peace took place. Since the flag is a symbol of speech, expressive conduct with regard to it must be speech. The second motivation is an expression of opinion by the state through legislation, an opinion that no one can contradict, which is unconstitutional. RAV v City of St. Paul p 1095 Teenagers made a cross out of chair legs and burnt it on the lawn of an africanamerican home. They were convicted under the St. Paul Bias-Motivated Crime Ordinance that forbid cross burning or nazi swastikas. Statute banning fighting words fails, teen is allowed to burn cross on lawn of africanamerican family's house. Government can't proscribe hate speech on the basis of its viewpoint (like race, gender or religious intolerance), but only in the way in which "fighting words" are expressed. Abusive invective does not fall under any of these categories. "Fighting words" are not speech, they are like noise from a truck, so they have no claim on 1st amendment, the city can prohibit them constitutionally. It is the manner in which the words are delivered that is prosctibable. Here, however, Minnesota is prosctibing fighting words on the basis of viewpoint (race/hate speech), and that is viewpoint discrimination. You cannot have viewpoint discrimination within the content of fighting words.Fighting words are words that by their very utterance incite a breach of the peace. Hurt feelings don't count, like in this case, only probability that they incite a breach of the peace. ANALYSIS for legislation that affects speech: a. Is the conduct expressive? a. Conduct was expressive in all cases. b. If legislation only incidentally affects speech in pursuit of a substantial government interest and is not viewpoint based/content based, it only receives intermediate scrutiny under lenient O'Brien test: EXAM: 4 part "O'Brien" test for intermediate scrutiny, only for content neutral laws. Statute receives this more lenient standard if: 1. The ends of the legislation are within constitutional power of government (police powers)

2. If it furthers an important or substantial government interest In O'Brien, national security, ability to raise an army quickly and communicate to draftees. 3. If the government interest is unrelated to the suppression of free expression a. If it is related to free expression, then scrutiny is higher under Johnson Here, the government interest is unrelated to the suppression of free speech because the reasons are administrative efficiency, a fast and efficient way of raising an army for changing circumstances etc. 4. If the incidental restriction on alleged First Amendment freedoms is no greater than is essential to the furtherance of that interest. c. If legislation is viewpoint based and does not pursue a substantial government interest it receives strict scrutiny under Johnson: a. If legislation is aimed at "fighting words," then it's legitimate, so long as it's not viewpoint based bans on "fighting words" (hate speech as fighting words). more here

ii. Does Money Equal Speech? Buckley v Valeo p 1101 Act limited individual contributions to political campaigns to $25k per year, or $1k per candidate per year. Statute establishing contribution ceilings unconstitutional, money is speech. The fact that a communication depends on the expenditure of money does not make that communication conduct, rather it is a form of constitutionally protected speech. Expenditure of money on political ads is speech because today's large society necessitates the expenditure of money for virtually any communication to the electorate. Although here speech is not restricted on the basis of content, it is restricted on the basis of who is speaking. Specifically it looks to equalize the volume of all voices in the political debate and to restrict the range of that political debate. The government can place time, place, and manner restrictions on demonstrations and other speeches (like the sound truck case). However, here, the government seeks to place restrictions on which political parties and interests have their voices heard in addition to time, place and manner restrictions because it aims restrictions "relative to a clearly identified candidate." A. Contribution limitations are constitutional because the Act tries to keep small voices from being drowned out by large contributors by putting a cap on individual contributions. The weighty interest of this portion of the Act outweigh the effect upon the First Amendment. B. Independent expenditure ceiling is also unconstitutional for the same reasons. Note that O'Brien does not apply here because the government's interests are legitimate, i.e. to ensure that voices are not drowned out.

Also, this is not a Ward "time place manner" restriction because it has a hard limit on contributions, which is not like a time place manner restriction. EXAM theory of 1st amendment: The concept that government may restrict the speech of some elements of our society in order to enhance the relative voice of others is wholly foreign to the First Amendment, which was designed to secure the widest possible dissemination of information from diverse and antagonistic sources and to assure unfettered interchange of ideas for the bringing about of political and social changes desired by the people. The interest of equal political competition is not enough. Cost of skyrocketing campaign costs is not sufficient either. McConnell v Federal Election Commission Campaign reform act of 2002 seeks to limit the influence of moneyed interests upon politics. "Hard" money is money donated to federal campaigns, while "soft money" is money donated to influence state and local campaigns. However, the purpose of "soft money" was expanded to "get out the vote" efforts and other activities that influenced national campaigns. Soft money could also be used to air "issue ads," or ads that did not use magic words like "Elect John Smith." 323(a) tries to take soft money out of federal politics with restrictive language. Restriction is constitutional, limit on soft money stands Money is not speech, it is property, and Congress has a right to protect the political system from the undue influence of money. P's arg is that forbidding "electioneering communication" is overly broad because it ignores the difference b/w issue advocacy and express advocacy (issue advocacy is permissible under 1st amendment). Court counterargs by saying that that was statutory interpretation not a constitutional ruling. Also, it is too easy to avoid that distinction with soft money b/c issue advocacy can easily be turned into express advocacy even if avoiding the "magic words."

Wisconsin Right to Life v Federal Election Commission p 1123 Section 203 of the Campaign Finance Reform Act makes it a crime for any corporation to to broadcast any communication regarding the name of a federal candidate shortly before an election. An ad is the functional equivalent of express advocacy, and therefore proscribable, only if the ad is susceptible of no reasonable interpretation other than as an appeal to vote for or against a specific candidate. Ban on issue ads months before an election is unconstitutional, only express advocacy bans are constitutional. So if the only reasonable interpretation of an ad is that it encourages people to vote for or against someone, then that ad is the functional equivalent of express advocacy and therefore proscribable. Problem is that an ad can have more than one reasonable interpretation even if it's clear. Bottom line: Buckley says money is speech and not proscribable, WRTL says that express advocacy ads can be limited months before an election but not issue ads, while McConnell on the other hand says that money is property, not speech, and Congress can pass legislation to keep it out of the election system. These cases probably cannot be reconciled, but can only be explained away by the ebb and flow of the personnel on the SCOTUS. Now that there are five concervatives, Buckley will probably prevail and allow money in elections, and McConnell is probably overruled (conservatives want money in politics, and Buckley allows money in politics).

McConnell Money is property, and it can be kept out of politics

Buckley Money is speech and cannot be proscribed

WRTL Express advocacy can be banned before an election, but not issue advocacy

There's one more case, Davis, but it's in the supplement, check notes if you need it. b. What Is an Abridgement of Freedom of Speech? i. Does the First Amendment Ever Protect Speech That Is Tortious? Defamation is the tort that envelops libel and slander. Both are false statements that may harm the reputation of another, libel being written and slander more transitory like spoken. NY Times v Sullivan Down south segregation was going on, and an ad in the NY times said that African American children were trying to exercise their rights under the constitution, and they were being prevented, and it solicited funds to support their right to vote and to defend MLK in a perjury suit. It then described an event where the police of Montgomery, AL padlocked black students in a school to keep them from eating. Sullivan is a police commissioner, and believed that"police" referred to him. He made no showing of pecuniary loss. Untrue speech by NY times against state is protected, NY Times wins. Private speech against government actors that is negligently false is protected, except if that speech is made with actual malice, meaning that the private actor knowingly makes an untrue statement. Negligently untrue statements against the government are protected so as not to chill free speech. NYTimes v Sullivan was about protection of speech against public officials, while Butts and Walker extended NY times protection to speech against "public figures." So public figures and public officials are different things, but they get the same Sullivan rules for speech against them. Public figures are intimately involved in addressing public questions or, by reason of their fame, shape events in areas of concern to society at large. Public figures get less protection because they have greater access to the limelight, and they have also assumed the risk of public debate and criticism. Gertz v Robert Welch p 1134 Newspaper published an article calling the P a marxist communist. P was a lawyer who is suing a police officer who shot him. The general picture the magazine painted of P was false. This case regards a publisher's constitutional privilege against liability for defamation of a private citizen. These were private persons, but the matter was one of public concern. Magazine can't claim first amendment, lawyer wins. States have the latitude to impose their own libel laws among individuals, no first amendment implications. Public figures have more of an opportunity to correct false statements because they can more easily grab the spotlight, but private individuals are more vulnerable.

Public Official/Public Figure Matter of Public Concern No recovery absent actual malice (Sullivan, Butts, Walker)

Private Person Recovery, no need to show actual malice for compensatory damages. Need to show actual malice for punitive damages (Gertz) Recovery, no need to show actual malice for compensatory or punitive damages. (Greenmoss)

Not matter of No recovery absent actual public concern malice (Sullivan, Butts, Walker), same as above

Matter of public concern/not matter of public concern matters only when a suit is brought by a private citizen, but it makes no difference when talking about suits brought by government officials. ANALYSIS for whether something is matter of public concern: There is no fixed standard for determine what is a matter of public concern. "Determination is fact dependent. It must be determined by content form and context of the statement revealed by the whole record." We have Gertz and Dunnen. Dunnen Letter published info on a company that it went bankrupt, and it went to five people who vowed to keep it private. It turned out to be false. The statement that this company went bankrupt is not a matter of public concern, it's related to that limited business audience. Gertz Gertz was a lawyer who represented a family who was suing a Chicago police officer for wrongful death. Newspaper called Gertz a communist who sued Chicago police department to undermine that department. This was a matter of public concern b/c it relates to the Chicago police department. This is the flavor of the distinction. Finally you do a POLICY analysis, which is that free speech must be robust and uninhibited, and wrong or false statements should be countered by other free speech. Would a blog be of public interest? It's published, but who reads it? Seems to be audience, and literally how many people are interested in it. Watch for when a private person becomes a public person, or a limited purpose public person.

ii. Is the Press Exempt from General Laws? The press is not exempt from general laws. Branzburb v Hayes p 1140

Courier-Journal story was about two defendants who told the reporter they made hashish and made $5k in three weeks. Journalist is subpoenaed to reveal the source of that information. Journalist loses, he has to disclose information. Newsmen are not exempt from the normal duty of appearing before a grand jury and answering questions relevant to a criminal investigation. Allowing the press a special privilege to circumvent a subpoena order to keep their sources confidential would require the court to define categories of reporters who may do this, which would be hard with bloggers and freelancers etc. Cohen p 1144 Minnesota newspapers went to P because P had some information on a gubernatorial candidate. He agreed to give the information in exchange for anonymity, which the newspapers agreed to. P's info was that the candidate had been convicted of two charges that were later found to be vacated and/or minimal. The newspapers published the source. P sued for promissory estoppel, saying the newspapers should have kept his name secret b/c they agreed to it. Newspapers said that if P won, then the judgment would be state action to quiet newspapers which would violate the first amendment. Contract was breached, newspaper loses, it should have stayed silent Generally applicable laws (promissory estoppel here) do not offend the First Amendment simply because their enforcement against the press has incidental effects on its ability to gather and report the news so long as the information was lawfully acquired. This case is the same as Branzburg because in both cases the press is curbed under generally enforceable laws. Here, the paper is breaking the promissory estoppel law, while in Branzburg, the journalist was breaking the law when he wasn't complying with a grand jury subpoena. This case was trickier b/c freedom of speech and the promissory estoppel that gagged the paper are closer conceptually, but it came out the same way b/c they were both generally applicable laws. However, if a generally applicable law is stretched to the point where it amounts to the chilling of speech in order to circumvent the Sullivan elevated standard, like intentional infliction of emotional distress under Falwell, it will be treated under Sullivan and actual malice will be required. Branzburg and this case also have in common the principal that the press is not allowed to decide what impedes the flow of information in the press, rather it is up to the courts. iii. May the Government Mandate Access to the Press? These cases are generally about the fairness doctrine. Red Lion A reverend attacked Mr. Cook, who wrote a book called "Goldwater - Extremists on the right." The Reverend made all sorts of allegations about Cook, calling him a communist etc. Cook wanted to reply to the allegations under the Fairness Doctrine, but the station would not allow him, so he sued and FCC found for him. Red Lion station is now challenging the constitutionality of the Fairness Doctrine. Fairness doctrine is constitutional, station has to give Cook airtime. The Fairness Doctrine requires that discussions of public issues by the media give both sides fair coverage. This rule is constitutional because it protects free speech by giving the little guy a chance to be heard in an age where the media has the loudest voice. Red lion is still good law, it's just that the fairness doctrine was abolished by FCC.

The media, which has access to the radio waves, is a small percentage of the population and they have an unfairly loud voice. The Radio Act gives them the right to use radio frequencies to the exclusion of others, but they do not have the right to monopolize those frequencies. OPINION: I think you can argue that since the government must have a hand in the distribution of wavelengths a priori, the government must also regulate it. Gov does not regulate the newspaper industry, but there is an FCC that regulates broadcast. Counterarg: you can allocate wavelengths through auctions like they do with cell phone wavelengths. He thinks the FCC is less political, they're the "good guys," and that justifies the fact that they play referree. Question: Do the FCC's fairness doctrine regulations, concerning personal attacks made in the context of public issue debates and political editorializing, violate the First Amendment's freedom of speech guarantees? Conclusion: In a unanimous decision, the Court held that the fairness doctrine was consistent with the First Amendment. Writing for the Court, Justice White argued that spectrum scarcity made it "idle to posit an unabridgeable First Amendment right to broadcast comparable to the right of every individual to speak, write, or publish." The Court held that the FCC's fairness doctrine regulations enhanced rather than infringed the freedoms of speech protected under the First Amendment. With respect to the regulation of personal attacks made in the context of public issue debates, the FCC's requirement that the subject of the attack be provided with a tape, transcript, or broadcast summary, as well as an opportunity to respond without having to prove an inability to pay for the "air-time," insured a balanced and open discussion of contested issues. The requirement that political editorializing be presented for and against both sides of the debated issues also contributed to the balanced discussion of public concerns. Tornillo State rep candidate in FL got debased by an editorial piece, and he demanded that his replies be printed in the newspaper verbatim under a FL statue requiring this. Equal space in papers is not required, newspaper doesn't have to print replies. The state cannot impinge on the function of a newspaper's editorial board and order a newspaper to print something that it does not want to print. The first amendment would be chilled because editors would steer clear of controversial topics lest they would be compelled to give complainers space (Sullivan). Newspapers are limited in their space, and if additional material would be required to be printed the newspapers would incur extra costs. This is not like the broadcast medium where paper is not involved. However, even if the costs were not high, statute would still be unconstitutional because the state would intrude on the editorial function of a newspaper.

iv. Is Some Speech Less Worthy of Protection? 1. Obscene and Sexually Explicit Speech Miller p 1158 D mailed pornographic materials to recipients who did not ask for them. D was convicted of mailing pornographic material. Obscene material is not protected, conviction stands, D loses Obscenity is not within the area of constitutionally protected speech. Obscenity is limited to the depiction of sexual conduct that appeal to the purient interest. A work appeals to the prurient interest if:

a. The average person, applying contemporary community standards (community standard, not national standards), would find that the work, taken as a whole, appeals to the "prurient interest" (community standard) a. What is the purient interest? It's not just any depiction of nudity, it's something that excites lust or a shameful or morbid interest in sexual matters. b. The work depicts sexual conduct in a "patently offensive" way (community standard) a. "Patently offensive" means hardcore, not softcore c. The work, taken as a whole, laks serious literary, artistic, political or scientific value (national standard). i.e. if it's for instructional purposes, like medical books, it's ok (however, this led to the ridiculous porn movies that start with "this is the sexual act"). Artistic value, however, is a national standard not a community standard, so SCOTUS gets to have the last word. What is "patently offensive" or appeals to the "prurient interest" is a community standard, not a national standard. This case overturns the "I know it when I see it standard," which did not provide certainty in the law.

Stanley p 1156 Police in GA found pornographic film in D's house when investigating allegations of bookmaking. He was convicted for the private possession of obscene material under a GA statute. Statute is struck down, D is allowed to watch porn in privacy The right to receive information and ideas, regardless of their social worth, is fundamental to our free society, especially when it is in the context of a person's private home. Here GA argues that it is trying to protect the D's mind from immorality, but that is in contradiction with the first amendment which says that states cannot control what its citizens think and read, especially in the privacy of their home. GA also argues that watching pornography may lead to immoral and criminal acts, but the court is not persuaded because the empirical data does not support that assertion. 2. Offensive Speech Ashcroft The Child Pornography Prevention Act proscribe dimages that appear to depict a minor engaging in sexually explicit activity whether that image was produced with an actual child or a computer graphic or an adult who looked like a child or any other means. Act fails as unconstitutional, computer graphic kiddie porn is ok The government may not bad speech meant for adults that is prohibited to children as a means to ban unprotected speech because that would be overly broad. Thus, the gov's argument that it's hard to tell apart real kiddie porn from fake kiddie porn for purposes of law enforcement must fail. Also, the gov's argument that child porn is bad for kids who are exposed to it is not a good argument. Child rapists can use candy to lure children.

With virtual kiddie porn, the argument that a child is harmed and cannot consent fails b/c no children are involved. Terminiello D, a Christian Veteran of America, spoke in a crowded auditorium, outside of which a large crowd protested. D was found guilty of disorderly conduct under a statute that prohibited speech that violates "public peace and decorum" as well as speech that "stirs the public to anger, invites dispute, brings about a condition of unrest or creates a disturbance, or it molests the inhabitants in the enjoyment of peace and quiet by arousing alarm." Statute unconst, protest allowed, D is released. Free speech may not be suppressed by the state because it leads to anger, unrest, public inconvenience, dispute or annoyance, but only if the danger rises to a clear and present danger of a serious substantive evil. In this case, only public inconvenience and unrest were caused, which is not enough. The test is whether the speech is likely to produce a clear and present danger of a serious and substantive evil. Cox 1500 africanamerican students protested segregation in LA by marching down to the rich white business district singing songs, clapping and sitting down at lunch counters to be served. Statute unconst, protests must be allowed A statute is unconstitutionally vague when its scope is overly broad and sweeps into constitutionally protected areas. Here, the first part of the statute banned the congregation with others "with intent to provoke a breach of the peace " While some breaches of the peace are not allowed, i.e. those involving clear and present evil, free speech may incite unrest and protest if it only amounts to a public annoyance. Thus, the statute sweeps too broadly into constitutionally protected areas. The state has a legitimate interest in maintaining public safety in the face of protests, but the power to allow or dissalow protests cannot be left entirely in the hands of officials. Public officials may only direct the time, place, duration and manner of the use of streets. Here, the LA statute gave officials discretion to allow or not allow a protest based only on the limited purpose of labor organization protests against low wages. All protests must be allowed, officials may only control the time and manner, not content of protest. That would be state control of speech. Cohen D wore a tshirt with the words "fuck the draft" printed on them. He was arrested in CA for disturbing the peace because the words were allegedly likely to incite others to rip his shirt off. Statute is unconstitutional, D is released. Where speech does not constitute fighting words because it is not directed at anyone, and it does not give rise to a clear and present danger, the speech is permissible. Here, "Fuck the draft" jacket obscenity can't be prohibited based on disturbace of the peace (because it did not disturb the peace), unwilling bypassers (because they can avert their eyes), fighting words (because they have to be directed to someone), it's not obscene (b/c it doesn't appeal to the prurient interest, not really sexually explicit, it's a political idea). Don't go around cursing at cops or judges because bad things are probably going to happen and you're not going to make a federal case out of it. So, speech can be proscribed if it:

Creates a clear and Constitutes fighting present danger of a words that are substantive evil. directed at someone But not if it: Sweeps away constitutionally protected speech because it is overinclusive (Cox) Creates only anger, unrest, dispute, annoyance or other public inconvenience

It is obscene under Miller, but limited by Stanley

Is done to restrict the time, place, duration and manner of the protest

Protected speech is banned as a way to ban unprotected speech (also overbroad, Ashcroft)

Ss socially worthless pornography (Stanley)

Looks like fighting words but are not directed at anyone, just written down or uttered mindlessly or randomly in the air (Cohen)

3. Commercial Speech Commercial speech is speech that proposes no more tha a commercial transaction of some sort. This is basically advertising, not speech made with a commercial purpose, or a charitable solicitation. Acai berry blog is on the edge. If it has links to places to buy, it's commercial speech, but if it's purely a blog, it's ok. Commercial speech is protected b/c it's in the public interest for consumers to have proper information about the products they buy.This is a right not just of the speaker but also of the listener. However, there's no protection for false commercial speech, unlike false political speech in Sullivan. If it's false, it's not protected. 10-1287 Village of Marvin --Summary judgment

Central Hudson test (applies only to commercial speech) for whether commercial speech is protected: The Public Service Commission of NYC ordered that the Electric Utility company stop running ads encouraging people to use electricity to heat their homes because the Commission found that the Utility does not have enough supply to fuel demands for the '73-'74 winter. Ban on ads was unconstitutional, Utility can keep running ads. There is a four part test, all of which must be met, that determines whether commercial speech is protected, or not: 1. The speech must be about a lawful activity and not be misleading. 2. The asserted countervailing government interest must be substantial

3. The regulation at issue must directly advance the governmental interest asserted 4. And the regulation must not be more expansive than it needs to be The fourth prong of the test is not met because a complete ban of advertisements for electricity is too extreme. The commission could have recommended changes to the ads to conform. Also, the statute is overly broad because it includes possible ads for other interests the Electric Utility may have, like public service announcements for the conservation of electricity. Liquormart RI statute banned the advertising of liquor prices except at the place where liquor is sold. Banning advertising of liquor unconst, liquor ads go back up. State legislatures may not impose blanket bans on truthful, nonmisleading information about a lawful product for paternalistic purposes. The statute will be reviewed with "special care" and is unlikely to survive scrutiny if its means are not narrowly tailored to its ends. Here, the state has not shown that banning advertising of prices will significantly reduce alcohol consumption market-wide. There are also less restrictive alternative means of achieving the same goal such as levying taxes on alcohol to make it more expensive. The "greater includes the lesser" argument that since the state can ban alcoholic beverages it can reduce its consumption by banning its advertisement does not work because a ban on a product is not "greater" than a ban on truthful speech. It's more injurious and intrusive to ban the teaching of fishing than to ban the selling of fish. The first amendment does not make an exception to bans on "vices" because the label "vice" can be liberally applied by a legislature to mean anything. Lorillard Tobacco A MA act forbid the outdoor billboard adverisement of cigarettes and other tobacco products within 1000 feet of a school or playground. It also forbids the indoor advertisement of tobacco products at a height of under five feet. Speech is protected, tobacco ads can stay up. A state need not demonstrate that there are no incursions into protected speech, but a speech regulation cannot unduly impinge on the speaker's ability to propose a commercial transaction and the adult listener's opportunity to obtain information about products. Here, the statute was not narrowly tailored because the geographic ban excluded tobacco ads from over ninety percent of Boston. Also, most places that sell tobacco cannot put too many ads in the window for security concerns, so the ban was not well thought out. Sales practice was upheld though b/c it was narrowly tailored. Info has to get to adults. Important test: If there are ample alternative channels of communications to get the message out, then the rule will be upheld. Rules are usually struck down on prong 4 b/c they're not narrowly tailored, and when they're passed they pass on prong 4 b/c there are ample channels of communication. A regulation cannot be sustained if it provides only ineffective or remote support for the government's purpose, or if there is little chance that the restriction will advance the government's goal. Forbidding the placement of ads at heights under 5 feet does not advance the government's goals because not all children are under five feet tall, and short kids can tilt their heads up and read ads at any height. EXAM: know your commercial speeches and your four part Hudson test.

ANALYSIS for commercial speech: There is a four part test, all of which must be met, that determines whether commercial speech is protected, or not (note that if it's content/viewpoint based it gets struck down under Johnson): 1. The speech must be about a lawful activity and not be misleading i. You can't advertise for illegal jobs, and you can't lie because false advertising is not protected 2. The asserted countervailing government interest must be substantial 3. The regulation at issue must directly advance the governmental interest asserted i. In Liquormart, the statute failed on this prong because banning liquor advertising did not significantly reduce alcohol consumption market-wide. 4. And the regulation must not be more expansive than it needs to be i. In Central Hudson, the statute failed on this fourth prong because the ban on ads was not narrowly tailored b/c it was too extreme since the commission could have recommended changes. It was also overly broad because it covered ads for other legitimate concerns the electric company had.

4. Adult Entertainment Renton City of Renton, WA passed an ordinance that bans adult movie theaters within a 1000 foot radius of any residential zone, single or multiple family dwellings, churches, parks or schools. Ordinance is constitutional, theaters are shut down. Regulations that ban speech on the basis of its content violate the first amendment, while content-neutral time, place and manner regulations are acceptable as long as they are designed to serve a substantial governmental interest and do not unreasonably limit alternative avenues of communication. A statute is content neutral if its primary concerns are restricting the "secondary effects" of that speech and those ends are justified without reference to the content of the regulated speech. Here, the legislature was concerned with the crime and vice that accompanies such theaters, not the lewd content of the movies played at those theaters. Therefore it is a content neutral statute, and the appropriate standard of review is time, place and manner. The time place and manner test is whether the statute allows for reasonable alternative avenues of communication. Here, that test is met b/c the legislature's interest is to protect the quality of urban life, and because it was reasonable to rely on studies done in Seattle on the banning of adult movie theaters within those zones. On the reasonable alternative avenues of communication the court finds that 5% of the city is left for these businesses to operate, and it's of no moment that that area has no vacancies because there was no pretense on Renton's part. City of Erie Erie enacted an ordinance banning public nudity. Ordinance stands, nudes have to put clothes on.

If the statute is related to the suppression of expression, it's unconstitutional, but if it's content neutral then it falls within O'Brien. Here, the ban on public nudity is content neutral b/c it bans all public nudity, not nudity with a certain message, and therefore it must be analyzed under O'Brien. Like in O'Brien, where the statute banned conduct of burning registration cards, not the message behind that burning, this statute bans public nudity, not the message behind any one given instance of public nudity. A statute is not unconstitutional if it aims to reduce the secondary effects of a content neutral activity even if it has de minimis intrusions on expression. Here, the difference between a g string and complete nudity is not great, and there is very little expression in taking off that last stitch of clothing (like in Ward where the sound amplification had to be controlled by a city employee, regardless of what type of music it was). Since statute is content neutral, O'Brien test follows: O'Brien factors (see O'Brien class notes): a. Is the government regulation within the state's constitutional power to act? i. Yes, Erie has the right to enact legislation to protect public health and safety (police powers). b. Does the regulation further an important or substantial government interest? i. Yes, its goal is to eliminate the secondary effects of public nudity which Erie relies on the research Renton did, which is fine as long as the evidence is reasonably believed to be relevant to the problem that the city addresses. Boos v Barry DC statute banned the display of any sign within 500 ft. of a foreign embassy that brought that foreign government into "public odium" or "public disrepute." Also prohibited any congregation of three or more persons within 500 ft of a foreign embassy. Statute said: Statute is unconstitutional, placards can be displayed in front of embassies. A statute that prohibits a) political speech b) in public forums such as sidewalks c) and is content based is unconstitutional. Here, the speech is a) political b/c it concerns protests of foreign states, b) on the sidewalks in front of embassies c) and dissalows only negative messages, not messages that praise foreign governments. Content neutral statutes that restrict speech justify that restriction without reference to the content of the regulated speech. Renton In Renton, the movie content was not referenced, only the secondary effects of those movie theaters, i.e. crime, property values, protection of residential neighborhoods. Here, the justification was the indignation of foreign dignitaries, but the effects that speech has on a third party listener are not secondary effects, but primary effects, and cannot justify suppression. Renton objected to the crime associated with those theaters, irrespective of what they were showing on the screen. Therefore, the statute is content based and unconstitutional.

ANALYSIS for adult entertainment : y Does the legislation concern itself with expressive conduct? y In Erie the public nudity was not found to have a message, and the fact that all public nudity was banned meant that it did not target speech but the nudity itself, which was a secondary effect. y Is the regulation content neutral? If it's content/viewpoint based, it gets strict scrutiny under Johnson.

Secondary Effects Analysis for whether rule is content based or content neutral:If the rule targets the behavior that comes with the speech, and not the content of the speech, it is content neutral. y In Renton, the government was worried about the crime and blight that comes with adult theaters, so it was not content based. y In Boos, the statute looked at the effect on the hearer, i.e. the foreign dignitaries to determine whether placards were lawful, but effect on hearer is a primary effect, not a secondary effect. y If the statute is content neutral and only regulates time, place and manner of speech, the O'Brien test follows: 4 part "O'Brien" test for intermediate scrutiny, only for content neutral laws: 1. Are the ends of legislation within the constitutional power of the government? (police powers) In Erie, protecting against nudity fell under health and safety police powers (it usually does). 2. Does the legislation further an important or substantial government interest? In O'Brien, national security and the ability to raise an army quickly and communicate to draftees were substantial government interests. In Erie, the city's goal was to eliminate the secondary effects of public nudity. The city relied on the research Renton did, which is fine as long as the evidence is reasonably believed to be relevant to the problem that the city addresses. In Renton the legitimate interest was that shady characters gather at adult theaters, and in that case it was enough for the court. 3. Is the government interest unrelated to the suppression of free expression? In O'Brien, the government's interest is unrelated to the suppression of free speech because the reasons are administrative efficiency, a fast and efficient way of raising an army for changing circumstances etc (connects to prong 2). 4. Is the incidental restriction on alleged First Amendment freedoms no greater than is essential to the furtherance of that interest? (is it narrowly tailored?) y Are there ample alternative methods of communication? y In Renton, 95% of Renton is off limits to these theaters, which is fine with the court b/c it relies on studies done in Seattle on the issue, while in Lorillard 87% and 90% foreclosure of tobacco advertising in Boston was fine with the court, which means that the court does engage in some content analysis and values theaters less than tobacco advertising. In Pacifica, they were given only a time window, and that wasn't enough.
y

5. Electronic Media and Content Regulation FCC v. Pacifica P was driving around with his son when he flipped on the radio that spewed Carlin's filthy words list. He complained and the FCC took it under advisement and appended the complaint to Pacifica's file. It's not a criminal proceeding, it's an administrative adjudication. Statute is upheld, Carlin has to be bleeped. "Indecent" means language describing sexual or excretory activities and organs at times of the day when there is a reasonable risk that children may be in the audience.

The first amendment does not prohibit all governmental regulation that dependson the content of the speech. Content based restrictions are constitutional if they restrict speech that has the effect of force and creates a clear and present danger that brings about the substantive evils that Congress has the right to protect against. Here, the regulation is clearly content based, but that is not enough. For example, libel, fighting words, and yelling "fire" falsely in a crowded theater are content based but can be abridged because they have the effect of force and create a clear and present danger that bring about the substantive evils that Congress has the right to protect against. Obscenity is part of that carveout, so long as it does not express a political point of view or has other social values. Here, the court did not find that Carlin was expressing a political point of view, although that's debatable. At the end of the day the court allowed Carlin's language to be curbed because it was broadcast over the radio which intrudes into people's homes and is easily accessible by children of all ages. Factors to be considered: a. Time of day b. Content of the program and therefore composition of the audience c. Was it on the radio, tv, or closed-circuit transmissions? The pig belongs in the barnyard, don't let the pig enter the parlor. Bottom line: Pacifica was not a ban on Carlin's dirty words routine, the problem that it was in the middle of the afternoon when children could be listening. If it was published in a book, it would probably be ok b/c parents are in charge of what kids check out of the library. "Excessible to children" and "broadcast invades your home" are the two differences with Cohen, but two justices signed onto "offensive speech has less value." Ashcroft v ACLU The Child Online Protection Act (COPA) was enacted by COTUS and it outlawed material that the average person, applying contemporary community standards, would find that it was meant to appeal to the prurient interest, simulates child pornography, or taken as a whole lacks serious literary, artistic, political or scientific value for minors. Act fails, kids can watch porn A statute is overinclusive if it suppresses a large amount of speech that adults have a constitutional right to receive where less restrictive alternatives would be as effective. Where the restriction is content based, the burden is on the government to show that plaintiff's proposed alternative is not as effective as the challenged statute. Here, the P shows that filtering software can be just as effective. It can allow the adult to control the content on the receiving end, and it can be more effective because it may block pornography originating from other countries. The preliminary injunction was properly accorded therefore because there is a less restrictive alternative. Statue is narrow b/c software can be used to control where kids go or other less restrictive authority. Dissent says that parents can't be there all the time. Also, kids know about computers more than parents, so they can probably find a workaround. Note how the case turns on technological alternatives and not on case law. "No less restrictive alternatives" is the test, so court concentrates on alternatives. Scalia thinks this is like pandering porn on the street, which was never constitutional to ban. Stevens wants to make it administrative (fines) rather than criminal.

ANALYSIS for electronic media: Note that content based discrimination is constitutional with obscene material so long as it does not express a political point of view or has other social values. y Is the speech broadcast on TV or Radio or other intrusive media? y Is it broadcast at an appropriate time of day? y Is the content of the program expected by the likely audience? y How broadly is it broadcast, is it radio, tv, or something more closed like the internet or closedcircuit TV? y At the end of the day, how likely is it that the audience will be offended/that inappropriate language will reach children? In Pacifica, the program was found to be on during the day, and broadcast broadly, to the point where it was very likely that children would hear his words. y How much of the speech is suppressed, and are there less restrictive alternatives of curbing? y Is the whole program cancelled, or just words bleeped? y If a large amount of speech is suppressed, is there a less restrictive alternative that the P can propose? In Ashcroft v ACLU filtering software was less restrictive for internet porn, and the court argued it was more effective, and that was enough. c. Government Property and the Public Forum The conflict in this area of the law is the fact that the government plays the dual role as a regulator and a property owner. Property rights grant the government the right to exclude individuals, but that ocasionally collides with the right to speak freely. Schneider Ordinances banned distributing handbills in public parks, or you needed permission from the government. The ordinance is unconstitutional, leaflets can be distributed without gov permission. Streets have traditionally been known as public place where information is distributed, and the public inconvenience of littering does not justify a law that impedes this right to share information in public places. Here, the state oversteps its boundaries by requiring pamphleteers to submit their ideas to the police for approval before they are dispensed on the street. If littering were a concern, the state should fine those who litter. Pamphlets hold a significant place in our history for facilitating public communication against the government. Mosley A Chicago statute banned picketing during school hours within 150 feet of a school while school was in session, unless the picketing was about a teacher labor dispute. P was picketing about the school's black discrimination because the school had a black quota. Statute is struck down as unconstitutional, black picketing may continue. Where a law restricts expression based on the content of the message, that law will be reviewed under strict scrutiny. The state may only limit the time, place and manner of picketing in order to further significant government interests.

Here, the statute disallows all picketing except for labor picketing, which is not a narrowly tailored law. All peaceful picketing must be allowed because peaceful race picketing is the same as peaceful labor picketing. The argument that some picketing tends to be more violent than another must fail because it projects into the future and it is also content based. Ward Rock Against Racism is a rock organization that plays rock music with an antiracism message. They were playing in Central Park, but were excessively loud. A NY statute required that the sound amplification be handled by a state provided technician so as not to have to call the police to cut off the power. The city considered other methods like setting a max decibel level, but this was impracticable because decibel levels varied with foliage, wind, temperature and crowd size. Statute stands, rock band has to deal with a state supplied sound technician. A content neutral time place manner regulation does not have to be the least restrictive alternative to achieve the government's interest, but it must not be substantially broader than necessary to achieve the government's interest and it must leave open ample alternative channels for communication. Where a time place manner regulation is content based, it must be the least restrictive alternative to achieve the government's goal, which is a feature of strict scrutiny, and it may be defeated by a showing of an alternative that is equally effective but less speech restrictive. If a statute is not content neutral, then showing any method that is less restrictive is likely sufficient to defeat it, which is a feature of strict scrutiny. So with content neutral legislation, the government can choose a slightly less narrowly tailored method, but with content based legislation the method must be the least restrictive, least broad, most narrowly tailored. Here, the city technician method directly furthers the city's legitimate interest, and those interests would have been less well served without the city technician. It leaves ample alternative channels for communication because it does not ban any speech, just controls the manner. ANALYSISb for analyzing a content neutral time place manner statute: a. What is the government's interest? b. Is the statute narrowly tailored to serve a significant governmental interest? a. Are there alternatives that are less restrictive that achieve the same ends as the statute? b. How closely is the statute interest matched with the methods of the statute? c. Are there ample alternative channels of communication? Krishna v Lee Krishna is a religious organization that hands out fliers at airport terminals in order to raise funds for the orgalization. Regulation stands, Krishna can't pass out fliers at the airport There are three types of forums: a. Forums that have traditionally been held in public trust and used for expressive activity. In these forums, speech regulation must be narrowly tailored to achieve a compelling state interest (parks, sidewalks only basically). Here, the airport has not been traditionally used for expressive activity. b. A designated public forum is subject to the same limitations (public concert stages).

c. All remaining public property that is not traditionally available to the public is not meant for public performances. The government may restrict speech on such property more liberally. A public forum is not created by government inaction, rather it is created by government action. Simply because pedestrians are allowed on a property does not make it a public forum (courtroom). Airports have been designated for air travel, not for the facilitation of ideas, and airports have not traditionally been held in public trust for the purpose of speech. Airports are like courtrooms, which are also public places, but protests cannot take place there. (POLICY: I assume it's b/c otherwise public buildings would become useless if people could pamphleteer in the middle of trials). Passengers who are loaded up with bags cannot afford to alter their paths to go around solicitors because they may miss a flight. Traditional forums for public speech are probably limited to streets and parks. MY COUNTERARG: how can a forum become a "traditional forum employed for expressive activity" if courts allow only those forums that are already traditionally held for expressive activity to be used as such, and not any new ones. It seems like these forums have been grandfathered in and no new ones can be created. Another argument is that if the court's concerns are that leaflettersharrass and keep passengers from getting to their flight, a statute can be passed that says leafletters in airports can't harrass or hold up passengers, which would be a more narrow tailoring and would better serve the ends of the legislation. Burson v Freeman Petitioner was not allowed to pass out campaign materials within 100 feet of a polling place in Tennessee pursuant to TN code. Statute stands, P is barred from passing out campaign material within 100 ft. Some restricted zone around polling places is necessary to serve the interest in protecting the right to vote freely and effectively. Here, the statute is content based so it is subject to strict scrutiny, for which a state needs a compelling state interest to ovecome. Here, that right is voting, and TN can set up a boundary. The size of the boundary must be reasonable and must not impinge on other rights, and not surprisingly the SCOTUS does not give us a number, but 100 feet is within constitutional boundaries.

Hill v Colorado CO passed a statute making it unlawful to approach a person within eight feet without consent to pass a leaflet or engage in oral protest, education, or counseling within 100 feet of the entrance to any health care facility. Petitioners protested abortion, and patients had to be escorted out of the hospital because of it. This "sidewalk counseling" was not confrontational. Statute stands, leafletters must stay away from patients The right to free speech includes the right to attempt to change another's point of view and may not be curtailed b/c speaker's message is offensive, but a listener must be allowed to avoid it. A regulation is content neutral if the government proscribes speech not because of disagreement with the message it conveys. Here, the statute is content neutral because it regulates the place where speech can take place, and the statute barrs all leafletting regardless of viewpoint. The regulation decides whether speech can occur, not what type of speech can occur. It also prohibits knowingly approaching a person, not speaking to the person (unlike in Boos where court looked at content of placards in embassy protests to determine whether foreign dignitaries would be offended).

Here, a court will look to the content of the leafletting to assess the purpose of the protester, but that is not a content based statute b/c courts always look at content to determine purpose. A regulation is content neutral if it does not discriminate among viewpoints. Here, the CO statute only placed a minor place restriction on an extremely broad category of communications with unwiling listeners. "Leafletters" includes any category besides abortion protesters under this statute, and that's the saving grace. Dissent disagrees that this statute includes those besides anti-abortionists. A regulation is content neutral if it is narrowly tailored to serve compelling state interests and leaves open ample alternative channels for communication. Here, the 8 foot rule does not impede patients from reading protesters' signs. If there were a limit on font size, that would be impermissible b/c their message would be completely suppressed and no alternative channels would be open. Also, in MA, there was a similar law that was struck down b/c it forbid approaching anyone coming out of the hospital as opposed to patients, which was too broad b/c we might want protesters to reach staff. Also, the statute would not be narrowly tailored if there were a complete ban on leafletting like there was a ban in Krishna. Unlike the airport terminal in Krishna, however, sidewalks are a traditional place of protest.

ANALYSISb for forums: What type of forum is it?  Three types: y Is it a traditional public forum?: y Streets, sidewalks and parks that have traditionally been used for the dissemination of information since time out of mind (Schneider). These forums are limited because it takes a long time to develop a tradition, and airports have not gotten there yet (point about grandfathering). y Is it a designated public forum?: y The library meeting room, concert venue (Ward). The only restriction is that you have to book a week in advance or what have you. y Note that in designated public forums, content based discrimination is allowed depending on the purpose of the designated public forum, but viewpoint discrimination is not allowed (unlike in traditional public forums where neither viewpoint discrimination nor content based discrimination is allowed). y If it is one of the public forums, is the regulation content neutral or content based? To figure out if it's content neutral or content based, ask: y Was the statute enacted because of disagreement with the message being conveyed? y In Johnson, the flag burning againts Reagan was clearly banned because of disagreement with the message, so it was struck down. y Can the statute be justified without reference to the content of the speech? y In Ward, the city technician can be justified because rule applied to any music to make sure it's not too loud for the neighbors. y Hill was a close case, but at the end of the day ban on leafletting by the hospital was justified to protect sick people from harrassment.

Does the statute base its ban on the source of the speech? y For example, only teachers can protest, not africanamericans. This is viewpoint discrimination, and it is never allowed in designated public forums or traditional public forums If statute is content neutral regulation of speech in a public forum, the test is under Ward: y Is the statute narrowly tailored(shouldn't be substantially broader than necessary to advance the governmental interest, it's ok if it's not the least restrictive alternative) to achieve a significant governmental interest? y Use Hill as analysis example (extending leafletting ban to anyone coming out of the hospital would be too broad). y Does it leave open ample alternative channels of communication? y Use Hill as an analysis example (font restrictions would close alternative channels) If statute is content based, it receives strict scrutiny y Is the statute narrowly tailored(must be the least restrictive alternative) to serve a compelling (Not "significant" like for content neutral. Compelling government interest is the prevention of voter fraud for example, Freeman) governmental interest.

If forum is public, is it reasonable to use the public property for speech? (asking whether it's "reasonable" to use it for speech is a broad test, but that's the test) a. Does it interrupt business? Being asked for money is too much, but flyers are ok b/c you can take them and walk away.

d. Government Speech and Government Support of Speech Rust v Sullivan A federal statute granted federal funds to family planning efforts at the state level, but not where abortion was a method of family planning. Petitioner argues that this amounts to "viewpoint discrimination" because statute forbids discussions about abortion in family planning. Statute stands, funding is ok even if it doesn't fund abortion advice. The government may selectively fund certain activities it believes are in the public's interest and not others without discriminating against a viewpoint and violating the constitution. Court finds that Title X funding requirements are not an unconstitutional restriction on the states because these facilities are not prohibited from suggesting abortions, just not under Title X. This case is like the Alaska case where the government was a party to a contract and could discriminate. Where the government uses its own money, it can promote its own points of view, or where the state is the speaker, it may make content-based choices. (Widmar) POLICY: If all government funding decisions could be subject to this scrutiny, then anyone could sue claiming that when the government funded one action it discriminated against another action. Rosenberger v University of Virginia

University of Virginia assesses a $14 fee to all students to pay for student activities. The student activity in question is one that seeks to foster an atmosphere of tolerance of Christian viewpoints. The Student Activities Fund refused to pay for the publication of a newspaper that had a Christian viewpoint. Requirement to pay fee is constitutional, Christian newspaper gets the funding. Where a state creates a forum for limited speech, content based discrimination is permissible if it preserves the purposes of that limited forum, but viewpoint discrimination is not permissible if the content of the speech is within the forum limits. The court did not rely on Rust because here, the University is a designated forum, and the speech comes from students, whereas in Rust the speech comes directly from the government. This is the public/private distinction, the fact that the speech is not university speech, but student speech, breaks the nexus with government. Again, this case is not like Rust because the government is not speaking directly through funding, rather it is funding a university whose purpose is to broadly support the voices of its students. If it did not fund voices equally, undergraduates like Marx, Russell and Sartre may not have their essays published. "Plato could contrive perhaps to submit an acceptable essay on making pasta or peanut butter cookies, provided he did not point out their imperfections." The forum here is the Student Activities Fund (metaphysical forum), and it disallowed material not because it was religious, but because it was presented from a religious editorial viewpoint. Viewpoint discrimination is an impermissible subset of content discrimination. Also, there is no Establishment Clause violation because so long as the University does not discriminate among viewpoints, then the requirements of the Establishment Clause are met. Pleasant Grove City v Summum The City of Pleasant Grove, Utah, installed monuments in a park that included 9/11 monument, a statute of the ten commandments, and others. Respondent asked the government to put up a statute containing "the Seven Aphorisms of Summum," but the city declined because it only put up statutes that had historical ties with the city. He won in lower court, so government appealed as petitioner. This is constitutional government speech, gov doesn't have to accept summum statue. Permanent monuments displayed on public property typically represent permissible government speech, even if they are privately donated and accepted by the government, so long as it is clear that the speaker is the government. Here, it's clear that the monuments are government speech. Also, just because an artist intended a certain message with a work of art, the fact that a government accepted the monument and displayed it does not necessarily mean that it adopted those points of view because a monument's message is open to interpretation. Governments need not be content neutral in their own speech because it is impossible to accommodate everyone's speech through public statues, and the government is entitled to its own point of view. Indeed it is the job of Government to set forth the people's point of view. If forum analysis leads to the closing of the forum, clearly forum analysis is not appropriate. The issue here is whether to apply Rosenberger or Pleasant Grove, i.e. whether it is government speech (and first amendment doesn't limit it) or private speech where nexus is broken. Ex.: State license plates that say "choose life" can be bought for a fee. If it's private speech, you begin applying forum analysis, but if it's government speech you stop and say first amendment doesn't apply.

Bottom line: Government can refuse to subsidize private speech, but it also cannot penalized private speech. Rust: Government can have restrictions on how its own funds are used (in this case for counseling). Rosenberger: Here, the government created a public forum where students expressed private messages, so viewpoint discrimination knocked it out. ALSO NOTE THAT TEN COMMANDMENTS CASES WILL NECESSARILY INVOLVE BOTH SPEECH ISSUES AS WELL AS ESTABLISHMENT CLAUSE ISSUES. ANALYSISb for government speech a. Is it government speech or private speech? a. Government speech is the display of statues even if they are privately donated (Pleasant Grove), as well as refusing to fund private speech (Rust), but the government cannot engage in viewpoint discrimination where it has created a public forum (Rosenberger) b. If it's government speech, first amendment does not apply c. If it's private speech in a traditional public forum, neither viewpoint nor content based discrimination is allowed, see analysis above. d. If it's private speech in a designated public forum, no viewpoint discrimination is allowed, but content based discrimination is allowed, see analysis above. e. Government as Educator (schools) Hazelwood A St. Louis high school newspaper wanted to publish an editorial piece on student pregnancy, but the principal did not allow it for fear that although names were excluded, the persons were still identifiable by the context. Moreover, discussion about birth control was viewed as inappropriate. This is constitutional, principal can cut the articles. If an activity carries the school's imprimatur, (i.e. so long as the actions are reasonably related to legitimate pedagogical concerns), student discipline is constitutional. Here, although the newspaper charter vowed not to restrict free expression, they were developed within the school curriculum, so they were part of a supervised learning experience for journalism students. Therefore school officials could regulate the speech. Here, the principal could have argued that the students had not mastered the journalism lessons of protecting people's identities regarding sensitive issues. Moreover, he only cut the articles, he did not suspend the rest of the articles. POLICY: Educators are entitled to exercise greater control over student speech that is part of the curriculum to assure that participants learn the intended lessons, that speech is not inappropriate in a school setting, and that student speech is not erroneously attributed to the school. Morse v Frederick The Olympic Torch passed through Juneau, Alaska, and high school students were allowed to go outisde and watch. Petitioner was a high school student who put up a sign that said "BONG HiTS 4 JESUS," which the principal confiscated because he viewed it as advocating the illegal use of drugs. This is constitutional, the principal can take down the sign.

A principal may constitutionally restrict student speech at a school event when that speech is reasonably viewed as promoting illegal drug use. Note, however, that advocacy for the legalization of drugs is probably allowed. While petitioner claimed that the sign meant nothing and was only intended to get him on TV, a reasonable interpretation would be the support of illegal drug use, and sanctioning the sign would send a signal to other students that the school sanctioned drug use. However, nobody could reasonably believe that this bore the school's imprimatur (so why would the students?). Had the speech been political, it would have been protected under Tinker where students were allowed to wear black armbands protesting the Vietnam war. 2. Freedom of Association a. Compulsory Disclosure of Membership NAACP v Alabama Alabama required all out of state corporations to qualify in the state before doing business there, and the state atty general required the NAACP to reveal the names of all its Alabama members to pursue this license. This violates freedom of assembly, NAACP doesn't have to reveal names. The freedom to engage in association for the advancement of beliefs and ideas is a liberty assured by the due process clause of the fourteenth amendment. Compelled disclosure of affiliation with groups engaged in advocacy constitute an effective and unconstitutional restraint on the freedom of association. Here, the production order is a restraint on the freedom of association, because it makes it more likely that people won't join for fear of having their identities and addresses disclosed. NAACP was able to show that there was a burde on association here because NAACP was able to show that in the past, disclosure of membership led to economic harm as well as threats of physical violence and coersion. Once an organization shows that the government's action (here, requirement of membership disclosure) substantially burdens its right to associate, the burden shifts to the government to justify the infringement under the strict scrutiny standard. Alabama failed to carry that burden here. Buckley v Valeo State law required the disclsure of persons who made campaign contributions. This is constitutional under right of assembly, parties must disclose their sources of funding. Laws that encroach on first amendment rights that require disclosure of membership receive the strict scrutiny standard of review that can only be overcome by substantial state interests. Here, however, there are three stubstantial state interests: a. Informing voters of corruption and appearance of corruption b. It keeps candidates from taking large contributions from special interests that may affect that candidate's performance in office. c. Record keeping is necessary to monitor and enforce election regulations. The problem in this case is that candidates were not able to show that there would be a substantial burden on minor parties and independent candidates. They simply could not identify the problems, so it's a problem of proof . Nevertheless, constitutional protection is dialed back.

b. Compulsory Expression Abood v Detroit Board of Education Michigan set up a system for union representation of local governmental employees (public school teachers). Employees must pay the union for this service as a condition of employment. This is constitutional, employees must pay for collective bargaining. A government may not require an individual to make payments to causes she does not support as a condition of employment, unless those payments are for services that benefit her. Here, the dues that must be paid go to a variety of causes that include but are not limited to collective bargaining. The government can compel the payment of union dues that go to collective bargaining, but not anything else, and the employment contract must give employees the option not to contribute to any other efforts. Board of Regents University of Wisconsin v Southworth U of Wisconsin charged fees for student extracurricular activities that expressed an opinion the petitioners disagreed with. Fees are constitutional, students have to pay the fees, university can fund activities. For unions, the constitution allows charging fees only for collective bargaining purposes and no more, but in the university setting the only limitation is that the university does not discriminate based on viewpoint. Same rule restated: Universities may sustain the extracurricular dimensions of its programs by using mandatory student fees with viewpoint neutrality as the operational principle. Fees for speech in the university setting cannot be limited because a court cannot limit the topics that students at a university may pursue as germane beyond requiring viewpoint neutrality in funding. The difference between this case and Rosenberger is that in Rosenberger the activity was already ongoing, here it was proposed to be initiated. c. Compulsory Association Roberts v Jaycees The US Jaycees is a non profit organization founded as the Junior Chamber of Commerce that gives young men guidance and opportunities for personal development. The St. Pauls chapter, however, began admitting women in violation of its bylaws, and the organization threatened to shut them down. The St. Paul chapter argues that doing so would be in violation of the Minnesota Human Rights Act that allows all individuals to enjoy goods and services without regard to race, sex, etc. The organization countersued claiming that requiring them to accept women is a violation of free speech. Minnesota Human Rights Act requiring the admission of women is constitutional, women are allowed. Freedom of association is constitutionally protected as as a fundamental element of personal liberty (personal/intimate association - right to come together) as well as a way to protect other individual liberties (expressive association - right to express as a group). Personal associations are relationships attendant to the creation and sustenance of a family, i.e. marriage, childbirth, the raising of children and cohabitation with relatives. Analysis points include size, purpose, policies, selectivity, congeniality, etc.

Here, none of the above features are present because only age and sex were relevant elements in the admission process, and each chapter had over 400 members which is not very cozy. Therefore, this is not a personal association and it should include women. The right of expressive association is the right to engage in a group effort to speak, worship and petition the government and the government may not impose penalties on group members, require the disclosure of the identity of an anonymous group member, or interfere with the organization of the group. Here, P charges that the government interfered with the organizaiton of the group, and the court agrees that this has taken place. However, the next step is to see if there is a countervailing compelling state interest, and there is. The countervailing compelling state interest is to protect women from stigmatizing injury and the denial of equal opportunities that result from rejection based on gender discrimination. Moreover, the state's solution is the least restrictive b/c the group already has women in it, and it's unlikely to change the viewpoint of the group. Boy Scouts of America v Dale The Boy Scouts assert that homosexuality is inconsistent with their values, and they excluded P, who is gay and a gay rights activist. NJ supreme court held that NJ's public accomodationstatutes requires that P be admitted. The NJ statute does not interfere with the Boy Scouts' right of expressive association, they can exclude homosexuals. Freedom of association presupposes a freedom not to associate. The forced inclusion of an unwanted person in a group infringes the group's freedom of expressive association if the presence of that person affects in a significant way the group's ability to advocate that group's viewpoints. However, that freedom can be overridden by regulations adopted to serve compelling state interests, unreltaed to the suppression of ideas, that cannot be achieved through means significantly less restrictive of associational freedoms. ANALYSIS for expressive association: a. Does the group engage in expressive association? Look to charter/purpose of organization. Here, yes it's an expressive association b/c its purpose is to instill values in young people. b. Does the inclusion of the new member significantly affect that group to advocate its viewpoint? The Boy Scouts believe that engaging in homosexual sex is not "morally straight" nor "clean." Courts can't judge the group's message, only to evaluate whether the group sincerely holds those beliefs, and it's clear that here they do. Dale, the P, is a gay activist whose beliefs are in direct opposition with those of the Boy Scouts, and if the Boy Scouts accept him they would send a message to the world that they accept homosexuals against their Oath, and that would contradict their message. c. Does the application of the given statute impose a significant burden on the group's freedom of expressive association? Here yes b/c the NJ law is very intrusive. It includes not only public and quasi public places like streets and restaurants, but also private groups such as the Boy Scouts. Dissent: Stevens, Souter, Ginsburg, Breyer: Appying Jaycee, the right to expressive association is limited by the compelling state interest of antidiscrimination based on gender. Sexual orientation should be treated in the same way.

Also, the majority should not have simply believed the plaintiff when he asserted in his brief that homosexuality is not morally straight and unclean. ANALYSISb for freedom of association: a. Is the government trying to interfere with first amendment right to associate by: a. Requiring disclosure of membership lists? a. If yes, the plaintiff has to show that the disclosure requirement imposes a substantial burden: a. In NAACP, the plaintiff carried this burden because in the past, mandatory disclosure of its members led to economic harm against those members, as well as threats of physical violence and coersion. b. In Buckley, the plaintiffs were not able to show that disclosure of persons who made campaign contributions were a substantial burden on independent candidates and minor parties, although the standard would probably be relaxed for them. b. If plaintiff carries her burden, the burden shifts to the government to show that there is a substantial state interest under strict scrutiny. a. In NAACP, the state was not able to show any good reason for the requirement, and if it did, nobody believed that it was not for racist reasons. b. In Buckley, this point was not reached. b. Requiring payment as a condition of employment or enrollment? a. Does the payment go to services rendered to plaintiff? a. In Abood, fees that were required for collective bargaining of city employees were constitutional b/c plaintiff took advantage of those services, but not the other fees. b. In Southworth, the university used the fees to sponsor activities that the plaintiff disagreed with, but there was viewpoint neutrality so the nexus b/w government funds and student speech was broken, and therefore fees had to be paid. c. Requiring organizations to allow members to join whom the organization is excluding? a. Is the association a personal association such as a marriage, childbirth, raising of children, cohabitation? a. Analysis points are the size of the association (the smaller the more personal), purpose (more family oriented more personal), selectivity (more selective more personal), congeniality (the "cozier" the more personal). The more personal the less the government will required compulsory association. a. In Jaycees, the association was too large (400 members) and the selectivity was low b/c only gender was taken into account, so it was not "cozy," and government could require the association to accept women. This makes it more of a public organization, and more susceptible to state statutes b. In Boyscouts, the group was more private, selection was more restrictive, and NJ law was more intrusive, so it was struck down. b. Is the association created for expressive association? a. Look to the charter of the organization. b. The boyscouts are an expressive association under Boyscouts, b/c organizaiton's purpose is to instill values in young people.

c. Is mandatory acceptance the least restrictive method that does not significantly affect the group's ability to advocate its viewpoint? a. In Jaycees, women were already allowed, so it was not a big inconvenience to require the admission of women. b. In Boyscouts, it was part of their creed that young men should be "morally straight," and that means no homosexuals. The court will not inquire into the message, only evaluate it for sincerity. P was a gay rights activits, and requiring that the Boyscouts admit him would interfere with their message. d. Is there an overriding state interest that defeats strict scrutiny? a. In Jaycees the interest of eliminating the bad effects of excluding women from organizations overrode any other interests. b. In Boyscouts, the interest of eliminating the bad effects of excluding homosexuals from organizations did not override any principles. 3. The Free Exercise of Religion a. Belief v. Conduct Reynolds In 1878, Reynolds, a mormon, was charged with bigamy because he took a second wife in the territory of Utah. Bigamy statute is constitutional under first amendment, mormons can't marry multiple wives. While the state may not constitutionally regulate belief, it may regulate conduct. This is necessary because otherwise any criminal action could be justified as a religious practice. A person could "sacrifice" a human being thereby circumventing murder statutes and acting according to his own set of laws. Religion cannot trump the law because the law is the supreme law of the land. b. Generally Applicable Laws Impeding Free Exercise of Religion Sherbert P was discharged because she was a seventh day adventist and would not work on a sabbath. She turned down a job so she was turned down for worker's comp because she refused to take suitable work. Statute fails, she gets benefits, doesn't have to work Saturday. Sherber balancing test: If the state imposes a substantial burden on the exercise of religion, it can only be justified through a countervailing compelling state interest under strict scrutiny. Does the statute impose any burden on the free exercise of the person's religion? Here, she was burdened because she was forced to choose to reject a precept of her religion in order to receive benefits, which is the same as fining her for her Saturday worship. The "baseline" must be a service that is offered to anyone, which is true of unemployment benefits. If so, is there a compelling state interest (not just rational) that justifies the infringement of the first amendment? Not here. Here, the state only provides that there would be an uptick in fraudulent unemployment claims by people who claim they can't work on Saturday. The court insists that the burden is on the state to show that there is no other alternative because this is a sensitive constitutional area that receives heightened review, and the state comes up short. Government must be neutral in the face of religion.

Smith P was a native american who was fired becaues of his ritualistic use of peyote for his religion. He could not obtain unemployment benefit because he was fired for good cause under the OR statute. The statute does not infringe on the first amendment, p can't get unemployment benefits. If the object of a regulation is not the prohibition or burdening of a religion, but it incidentally burdens religion and is otherwise neutral and generally applicable, then the regulation does not infringe on the first amendment. An Amish family cannot refuse to pay taxes because their faith does not allow them to support organized government. Here, the statute was generally neutral, and a religion was burdened only incidentally, so it stands. Sherbert applies only if a statute prohibits religious specific activities. For example, a statute that forbid wine drinking would not be unconstitutional, but a statute that prohibited sacramental consumption of wine would be unconstitutional. HYALEAH AND O'CENTRO ANALYSISb for laws impeding the free exercise of religion: Does the law impose a substantial burden on the free exercise of religion? a. In Sherber, law imposed a substantial burden b/c P was forced to choose between a precept of her religion (worshipping on Saturday instead of working) and employment. b. In Smith, smoking peyote instead of receiving unemployment benefits did not count because the law was generally applicable and non discriminatory on its face, and the banning of religious use of peyote was only incidental. c. An Amish family cannot refuse to pay taxes because their faith does not allow them to support organized government, because taxes are a compelling state interest and the Amish are only incidentally burdened. If so, is there a compelling state interest (not just rational) that justifies the infringement of the first amendment? a. In Sherber the reason that there would be an uptick in fraudulent unemployment claims was not sufficient. The burden is on the government to show there is no less restrictive alternative. b. In Smith, the criminalization of drugs worked.

Fundamental rights: y Voting (Harper) y Travel (Saenz) y Speech (O'Brien) Suspect classifications: y Wealth (if a person cannot afford something at all [Griffin, Douglas], or if a person does not have enough money to access a fundamental right [Harper] but not if a person does not have enough money to access something that is not a fundamental right, such as education [Rodriguez]) y Viewpoint based limits on speech (Johnson).

You might also like